ORTHOPEDIC MCQS OB 20 BASIC 1
-
Which of the following is the most accurate definition of stress shielding?
- The decrease in physiologic stress in bone due to a stiffer structure sharing load
- Electrochemical potential created between two metals in physical contact and immersed in a conductive medium
- Degradation from exposure to a harsh environment
- Physical movement of two plates against each other leading to mechanical wear and material transfer at the surface
- Bone death secondary to compromise in blood supply
Corrent answer: 1
Based on Wolff’s Law, bone in a healthy person will remodel in response to applied load. Orthopedic implants often reduce applied load, resulting in bone resorption, leading to decreased bone mineral density or osteopenia. This will be apparent on plain radiographs, and can be seen with arthroplasty implants or with fracture fixation implants.
ORTHOPEDIC MCQS OB 20 BASIC1
Incorrect answers:
2: This is the definition of galvanic corrosion. 3: This is a generic corrosion definition.
4: This is a definition of fretting.
5: This is a definition of avascular necrosis.
-
All of the following statements regarding RNA are true EXCEPT?
- RNA uses thymine while DNA uses uracil
- RNA may be either single or double-stranded
- RNA has a hydroxyl group at the 2' position in the ribose
- DNA has a hydrogen group at the 2' position in the deoxyribose
- RNA helix is of A-Form
Corrent answer: 1
All of the statement listed are true EXCEPT for "RNA uses thymine while DNA uses uracil" which is a false statement.
There are two types of nucleic acids: Deoxyribonucleic acid (DNA) and
ribonucleic acid (RNA). The DNA contains the genetic codes to make RNA and the RNA in turn then contains the codes for the primary sequence of amino acids to make proteins. The DNA bases are thymine, adenine, guanine, and cytosine (TAGC). RNA bases are uracil, adenine, guanine, and cytosine (UAGC). DNA is double-stranded, whereas RNA may be either single or
double-stranded. DNA has a hydrogen group instead of a hydroxyl group at the 2’ position in the ribose. DNA is only located in the nucleus, whereas RNA is found in the nucleus and the cytoplasm in the form of mRNA. RNA helix geometry is of A-Form whereas DNA helix is of B-Form.
Musgrave et al discuss important gene therapy and tissue engineering issues
in orthopaedic surgery. Specific areas of musculoskeletal medicine where gene therapy and tissue engineering have shown promise and early treatment success include the areas of bone healing, cartilage repair, intervertebral disk pathology, and skeletal muscle injuries.
Incorrect Answers:
Answer 1: False - DNA uses thymine while RNA uses uracil. Answer 2: True - RNA may be either single or double-stranded
Answer 3: True - RNA has a hydroxyl group instead of hydrogen group at the 2' position in the ribose
Answer 4: True - DNA has a hydrogen group at the 2' position in the deoxyribose
Answer 5: True - RNA helix is of A-Form
-
Medical treatment targeting TNF-alpha has revolutionized which of the following diseases?
- Osteoarthritis
- Rheumatoid arthritis
- Hunter syndrome (type II mucopolysaccharidosis)
- Hereditary vitamin D resistant rickets
- Gout
Corrent answer: 2
TNF-alpha medical therapy has revolutionized the treatment of rheumatoid arthritis.
DMARDs have revolutionized the treatment of rheumatoid arthritis, shifting the focus from treatment of symptoms to prevention of progression. Specific research into TNF-alpha modification has also shown progress in the treatment
of juvenile rheumatoid arthritis, seronegative spondyloarthopathies, chronic recurrent multifocal osteomyelitis, and histiocytosis X. There are many different types and mechanisms of actions of DMARDs. One large family of medications (including etanercept) functions through antagonist action against TNF-alpha. Other types of DMARDs are anti-folate (methotrexate), IL-1 inhibitors (chloroquine), or pyrimidine synthesis inhibitors (leflunomide).
Howe et al. discusses how to modify TNF antagonists in the perioperative period. They recommend for minor procedures the medication can be continued. For moderate to intensive procedures, they recommend withholding etanercept for 1 week, and plan surgery for the end of the dosing interval for adalumimab and infliximab, and restart 10-14
days postoperatively.
-
All of the following statements are true of osteocalcin EXCEPT:
- It is the most abundant noncollagenous protein of bone
- It is secreted by osteoclasts
- It is involved in mediating calcium homeostasis
- It has been used as a biochemical marker of bone formation
- It is part of the organic matrix of bone
Corrent answer: 2
Osteocalcin is the most abundant noncollagenous protein in bone. It is secreted by osteoblasts and plays a role in bone mineralization and calcium homeostasis. It is pro- osteoblastic, or bone-building, by nature. It has recently been used as a biochemical marker for bone formation in trials of anabolic drugs, such as forteo. Answer 2 is the false statement because osteocalcin is secreted by osteoblasts.
-
All of the following are clinical features of complex regional pain syndrome (reflex sympathetic dystrophy) of the lower extremity EXCEPT:
- Swelling
- Cool and shiny skin
- Allodynia
- Crepitus
- Hyperpathia
Corrent answer: 4
Complex regional pain syndrome is a complex clinical problem for the orthopaedic surgeon. In the acute stage (<3 months), burning or aching pain that cannot be controlled by narcotics is the major feature. Swelling, cool and shiny skin, allodynia and hyperpathia are also clinical features. Crepitus, however, is not a clinical feature.
Hogan et al provide a review of the evaluation and treatment of complex regional pain syndrome. They argue "despite many divergent and often conflicting theories, the cause of the severe pain, alterations in regional blood flow, and edema is unknown. Interventions that have proved successful for treating similar conditions in the arm and hand frequently do not relieve pain similarly in the lower extremity. Common treatment regimens target individual components of this symptom complex, namely, sympathetic or afferent nerve
hyperactivity, vasomotor instability, or regional osteoporosis. Despite widespread use of some of these treatments, few controlled clinical trials quantify their effectiveness."
Illustrations A and B shows characteristic features of complex regional pain syndrome of the lower extremity.
-
A patient sustains the fracture shown in Figures A and B. Compared with open reduction and internal fixation with a conventional, non- locking condylar buttress plate, what potential complication is more likely to occur if this fracture is treated with closed reduction and minimally-invasive, locking screw-only fixation with a distal femur locking plate?
- Non-anatomic reduction of the articular surface
- Soft tissue stripping
- Shortening
- Absence of visible callus
- Devascularization of fracture fragments
Corrent answer: 1
Articular fractures treated with closed reduction and minimally-invasive, locking screw- only fixation are at increased risk of non-anatomic reduction of the articular surface.
Closed reduction and locked plating of an articular surface is less desirable because articular congruity cannot be achieved except under direct visualization. This leads to non-anatomic reduction. On the other hand, locked
plates are well suited for periarticular fixation if direct reduction techniques are used. The fracture must be reduced and the limb alignment, length, and rotation must be set before placement of locked plates.
Egol et al. reviewed the biomechanics of locked and unlocked constructs. They found that compression plates provide anatomic reduction, facilitating primary bone healing (via endosteal healing) by reducing strain to < 2% (absolute stability). On the other hand, locked plates provide relative stability and with a wider gap and strain < 10%, allowing secondary bone healing through endochondral ossification.
Haidukewuch reviewed innovations in locking plates. They recommend the following indications for locked plates: indirect fracture reduction, complex periarticular fractures with metaphyseal comminution, such as distal femoral and proximal tibial fractures, fractures in osteoporotic bone, bridging comminuted fractures to minimize soft tissue damage, fractures with short periarticular fragments and periprosthetic fractures, plating of fractures when compression plating on the tension side is anatomically impossible.
Figures A and B show a comminuted intraarticular distal femur fracture. Illustration A shows a conventional condylar buttress plate. Locking screw options are not available for this plate. Illustration B shows a distal femur locking plate with aiming jig. Only locking hole options are available for this plate.
FOR ALL MCQS CLICK THE LINK ORTHO MCQ BANK
Incorrect Answers:
Answer 2: Closed reduction with locked plating places the plate in the submuscular plane and is respectful of soft tissues and preserves soft tissue attachments.
Answer 3: Distraction is the main method of indirect reduction when performing closed reduction and percutaneous plating. This allows restoration of anatomic length.
Answer 4: ORIF with absolute stability (compression plating, or lag screw fixation with neutralization) will result in primary bone healing and less callus compared with bridge plating with relative stability, which often results in marked callus formation.
Answer 5: Percutaneous locked plating preserves vascularity by minimizing soft tissue disruption, and also by minimizing plate contact with bone, preserving periosteal perfusion.
-
A 43-year-old female with long-standing rheumatoid arthritis complains of right forefoot pain for several years. She has failed conservative treatment and radiographs are shown in Figure A. What is the most appropriate treatment?
- Bunionectomy
- Keller procedure with lesser metatarsal head resections
- 1st MTP joint fusion and lesser metatarsal head resections
- Forefoot amputation
- 1st MTP joint interposition arthroplasty and lesser MTP joint arthroplasties
Corrent answer: 3
Surgical treatment of a rheumatoid forefoot involves fusion of the 1st MTP and lesser metatarsal head resections. The earliest manifestation of rheumatoid arthritis of the forefoot is synovitis of the MTP joints with eventual hyperextension deformity of the MTP joints including distal migration of the forefoot pad, painful plantar callosities and skin ulcerations over bony prominences.
Coughlin et al reported on the results of reconstruction of the rheumatoid forefoot in the manner described above. In 58 feet with an average of 6 year follow-up, the patient satisfaction rating was excellent/good for 45, and pain was rated as absent or mild in 43 feet. Most patients still have some activity limitations.
Illustration A shows the preoperative and postoperative anteroposterior
radiographs of hallux valgus deformity with lesser metatarsophalangeal joint erosions treated by 1st MTP joint fusion and lesser metatarsal head resections.
-
You are considering using a frozen allograft distal femoral condyle in your reconstruction of a massive giant-cell tumor of the knee. In counseling your patient regarding the risks of allografts, you explain that 5 years after transplantation, what percentage of donor chondrocytes will be present and viable in the allograft?
1. 1-5%
2. 6-20%
3. 21-50%
- greater than 50%
- None, by 5 years the allograft cartilage will be completely acellular
Corrent answer: 5
By 5 years, the allograft cartilage will be completely acellular, so there will be no residual donor chondrocytes.
Enneking et al. conducted both radiographic and histologic studies of sixteen massive retrieved human allografts four to sixty-five months after implantation. Analysis of the articular cartilage revealed no evidence that any chondrocytes had survived, even when the graft had been cryoprotected before it was preserved by freezing.
-
A 64-year-old woman with a longstanding history of rheumatoid arthritis complains of finger dysfunction for the past 6 months. Figure A displays her hand during active extension of all fingers. Figure B displays her hand maintaining her fingers extended following passive extension. What is the next most appropriate treatment for the ring finger?
- Spiral oblique retinacular ligament reconstruction
- Sagittal band reconstruction
- Lateral band reconstruction
- Central slip reconstruction
- Triangular ligament and transverse retinacular ligament reconstruction
Corrent answer: 2
Sagittal band disruption is often associated with rheumatoid arthritis. When this patient attempts to actively extend the affected digit, the extensor tendon
subluxates ulnarly as a result of the sagittal band rupture, and is left with an extensor lag. If one passively extends the finger fully, the patient is able to maintain this position, as the tendon is intact.
Sagittal band reconstruction can be performed with Watson's technique of creating a distally based tendon graft harvested from the central third of the extensor tendon, passed deep to the intermetacarpal ligament and sutured back to itself. Illustration A depicts an intraoperative view of the tendon before reconstruction and Illustration B displays tendon following sagittal band reconstruction. Illustration C displays all of the anatomic locations
of the options listed above.
-
A splenectomy is performed in a 7-year-old boy following a motor vehicle accident. All of the following are recommended for long-term management EXCEPT:
- Pneumococcal vaccination
- Haemophilus influenza type B vaccination
- Meningococcal group C vaccination
- Lifelong prophylactic antibiotics
- Hepatitis A vaccination
Corrent answer: 5
All of the responses are correct except the need for Hepatitis A vaccine. Hepatitis A is a virus with tropism for hepatocytes which causes infection from fecal-oral contaminated food/water, and shows no increased rate of either infectivity or morbidity in patients with hyposplenism.
Basic recommendations for splenectomized patients include:
- All splenectomized patients and those with functional hyposplenism should receive pneumococcal immunization.
- Patients not previously immunized should receive haemophilus influenza type B vaccine.
- Patients not previously immunized should receive meningococcal group C conjugate vaccine.
- Influenza immunization should be given.
- Lifelong prophylactic antibiotics are still recommended (oral phenoxymethylpenicillin or erythromycin). This is seemingly despite lack of good data demonstrating a role for lifelong chemoprophylaxis and the acknowledgement that long-term compliance may be problematic.
Davies et al review the current level of evidence supporting these guidelines
for infection prevention in patients with hyposplenism. New to these guidelines are issues regarding occupational exposure and the use of the meningococcal group C and the seven-valent pneumococcal vaccine in non-immunized hyposplenic patients.
Gandhi et al evaluated their nonoperative management of blunt splenic injury in pediatric trauma care. They found compared to historical controls, children with blunt splenic injuries who were hemodynamically stable could be safely monitored with a protocol which required 4 days of inpatient care, 3 weeks of quiet home activities, and 3 months of light activity. This protocol seems to allow for safe return to unrestricted activity.
Incorrect Answer:
Answers 1-4 are incorrect because they are recommended in splenectomy patients.
-
Ligaments attach to bone by both direct insertion and indirect insertion. Which of the following most accurately describes the order of the four transition zones of direct insertion?
- Ligament > fibrocartilage > mineralized fibrocartilage > bone
- Ligament > mineralized fibrocartilage > fibrocartilage > bone
- Ligament > mineralized fibrocartilage > periosteum > bone
- Ligament > Sharpey's fiber > periosteum > bone
- Ligament > periosteum > fibrocartilage > bone
Corrent answer: 1
There are two types of tendon/ligament insertion into bone: direct and indirect insertion. The more common, indirect insertion, occurs when the superficial ligament fibers insert into the periosteum. Direct insertion of tendon/ligaments
into bone occurs through a transition of 4 distinct phases: 1) ligament, 2) fibrocartilage, 3) mineralized fibrocartilage, and 4) bone.
-
While flexing the elbow to perform a biceps curl, what type of muscle contraction is occuring?
- Isometric
- Isokinetic
- Plyometric
- Eccentric
- Concentric
Corrent answer: 5
Concentric muscle contractions occur when a muscle shortens during contraction, as in the upward motion when performing a biceps curl. An eccentric contraction occurs when a muscle lengthens with contraction, as in the "negative" or lowering motion of a biceps curl. An example of an isometric (muscle contracts while maintaining constant length) contraction would be pushing against an immovable object. An example of an isokinetic (muscle has constant speed of contraction) occurs with specialized equipment like Cybex machines. Plyometric contractions occur when a muscle rapidly lengthens just prior to contraction - like during repetitive box jumping.
Woo and Buckwalter describe the mechanisms, barriers, and molecular processes involved in ligament and tendon injury and repair.
-
A 34-year-old laborer has her left foot crushed in a piece of farming equipment as shown in Figure A. All of the following are reasons for a poor outcome following a crush injury to the foot EXCEPT:
- Workers compensation injury
- Development of reflex sympathetic dystrophy (complex regional pain syndrome)
- Delayed soft-tissue coverage in mangled extremities
- Immediate skeletal stabilization
- Ongoing litigation
Corrent answer: 4
This patient has a mangled extremity. Rigid skeletal stabilization is recommended to enhance soft-tissue healing.
Level 4 evidence from Myerson et al found that delayed soft-tissue coverage in mangled extremities correlated with poor outcome. Poor results also occurred
if treatment was not immediately initiated (immediate debridement shown in Illustration A), if patients subsequently had neuritis or reflex sympathetic dystrophy, or if patients were involved in ongoing workers' compensation and litigation. Neuroischemia following substantial soft-tissue injury likely plays a role in the development of chronic pain after crush injuries to the foot, either through direct trauma to the peripheral nerves or by intraneural or extraneural fibrosis. This trauma to the nerve may cause chronic neuritis, which then triggers a sympathetically mediated reflex sympathetic dystrophy (complex regional pain syndrome).
-
A 65-year-old man undergoes total knee replacement and is found to have deep vein thrombosis two days later. What molecule is
thought to be involved in this process when it is released during surgical dissection?
- Prothrombin
- RANKL
- IL-1b
- Thromboplastin
- Factor XI Corrent answer: 4
Thromboplastin is also known more commonly as Tissue Factor (TF), which is involved in the Extrinsic Pathology of the coagulation cascade.
During surgical dissection, insults occur to the endothelial walls of blood vessels. There are three ways in which the body reacts to form a clot so that the patient does not bleed excessively. One is via vessel contraction, another is by collagen release, and a third is by tissue thromboplastin release. Thromboplastin release is part of the extrinsic coagulation pathway (see
Illustration A). Thromboplastin release activates Factor VII which activates Factor X which converts prothrombin to thrombin. Thrombin is the catalyst for converting fibrinogen to fibrin which induces clot formation. While this is useful for decreasing bleeding, it is the same mechanism by which a deep venous thrombosis (DVT) develops.
-
Which of the following materials has a Young's modulus of elasticity that is most similar to cortical bone
- Titanium
- Zirconia
- Stainless steel
- Ceramic (Al2O3)
- Alloy (Co-Cr-Mo)
Corrent answer: 1
Of the materials listed Titanium has an elastic moduli closest to cortical bone. Titanium is extra-ordinarily light, strong, highly ductile, and corrosion resistant. Titanium is however very notch sensitive and has poor wear resistance.
Young Modulus of Elasticity is defined as the stiffness (ability to maintain
shape under external loading) of a material. On the stress vs. strain curve it is defined as the slope of the line in the elastic zone (see Illustration A). Young’s modulus is constant and different for each material. The relevant moduli (unit GPa) are approximated below:
1) UHMWPE = 0.8-1.5.
- Cancellous Bone = 2
- PMMA = 3.1
- Cortical Bone = 18
- Titanium = 115
- Tantalum = 186
- Stainless Steel = 240
- Cobalt-Chromium Alloy = 240
- Zirconia (Ceramic) = 248
- Alumina = 340
Illustration A shows a stress vs. strain curve. Young Modulus of Elasticity is defined is defined as the slope of the line in the elastic zone
-
A prospective cohort study is performed looking at the relationship between blood transfusions and the risk of developing hepatitis C. In the transfused group (study group) of 595 patients, 75 patients develop hepatitis C. In the non- transfused group (control group) of 712 people, 16 people contract hepatitis C. What is the relative risk of developing hepatitis C with a transfusion.
- Incidence of study group (75/595) divided by incidence of control group (16/712)
- Incidence of study group (16/595) divided by incidence of control group (75/712)
- Prevalence of study group (75/595) divided by prevalence of control group (16/712)
- Prevalence of study group (16/595) divided by prevalence of control group
(75/712)
- Total infected (75+16) divided by total population in study (595+712) Corrent answer: 1
Relative risk is the risk of developing disease for people with known exposure compared to risk of developing disease without exposure. It is calculated by dividing the incidence of the study group by the incidence of the control group.
The cited Kocher article details commonly used statistics in orthopedic research.
-
In a patient undergoing total knee arthroplasty, the femoral and tibial bone resections can be done using intra-or extra-medullary alignment systems. Extra- medullary guidance systems have what benefit over intra-medullary guidance systems?
- Decreased fracture risk
- Decreased embolization risk
- Decreased surface area available for cement interdigitation
- Increased risk of blood loss and/or transfusion requirement
- Increased rates of cerebral ischemia
Corrent answer: 2
Compared to intra-medullary guidance systems used for total knee arthroplasty, extra- medullary guidance systems show decreased embolization risk for the patient. The reason for this finding is that embolism syndrome in TKA typically is due to the placement of intra-medullary instrumentation for determination of the anatomical alignment.
Kim et al prospectively showed that fat embolization occurs due to intramedullary insertion of the alignment components. In their study, fat embolism was more common in bilateral than unilateral TKA as 65 patients (65%) with a bilateral TKA compared with and 46 patients (46%) with a
unilateral TKA developed fat emboli.
In a second study, Kim et al evaluated the risk of fat embolism associated with computer navigated total knee arthroplasty and found that computer
navigation does not appear to impart an elevated risk of fat embolism to the patient.
Ries et al studied the effect of different alignment rods on the risk of fat embolism. They found that intramedullary pressure during alignment rod insertion was significantly higher in patients with round compared with fluted alignment rods. As such, they recommend a fluted rather than a round intramedullary alignment rod to be used to minimize intramedullary pressure and pulmonary shunting during knee arthroplasties.
-
Vitronectin is an important receptor involved in which of the following functions?
- Interacts with RANK Ligand to stimulate osteoclasts
- Osteoclasts attaching to bone
- Competitive inhibition of RANK Ligand
- Chemotaxis in fracture healing
- Type II collagen formation
Corrent answer: 2
Vitronectin, integrin avß3, assists osteoclasts to attach to bone.
Osteoclasts attach to bone surfaces by means of integrins and then seal the space below. A ruffled border is then created and bone matrix is removed by proteolytic digestion through the lysosomal enzyme cathepsin K.
Incorrect Answers:
Answer 1: RANK is a receptor on osteoclasts that when activated by RANKL stimulates osteoclasts. RANKL is found on osteoblasts.
Answer 3: Osteoprotegerin (OPG) decreases osteoclast differentiation by it’s interaction with RANKL (receptor activator of NF-kappaB ligand). OPG is made by osteoblasts and binds to RANKL (RANK Ligand) to competitively inhibit RANK binding.
Answer 4: PDGF (Platelet Derived Growth Factor) is involved in fracture healing. It is chemotactic and attracts inflammatory cells to the fracture site and is important in early fracture healing, especially the hematoma formation. Answer 5: TGF-B (transforming growth factor Beta) induces mesenchymal cells
to produce type II collagen & proteoglycans. It is important in the early stages of fracture callus formation.
-
Which of the following is a phenomenon whereby the symptoms of a genetic disorder become apparent at an earlier age as it is passed on to the next generation?
- Genetic drift
- Expansion
- Mendelian inheritance
- Anticipation
- Phenotypic plasticity
Corrent answer: 4
Anticipation is a phenomenon whereby the symptoms of a genetic disorder become apparent at an earlier age as it is passed on to the next generation. In most cases, an increase of severity of symptoms is also noted. Anticipation is common in trinucleotide repeat disorders such as Huntington's disease, myotonic dystrophy, Friedreich ataxia, and Fragile X syndrome. Illustration A shows the genetics of the trinucleotide repeat disorders.
-
A 58-year-old female complains of continued pain and swelling 6 months following total knee arthroplasty. She describes a burning pain that radiates from the knee down the anterior compartment of the leg. The pain arises sporadically and is associated with swelling,
sweating, and a purplish hue of the leg. Knee radiographs are
provided in Figures A and B. Aspiration is negative for infection. Which of the following is the best management?
- Lumbar spine MRI to evaluate for radiculopathy of the L3 nerve root
- Alpha-adrenergic blockers, physical therapy, tactile discrimination training, and graded motor imagery
- Surgical exploration of the knee
- Surgical debridement, pulsatile irrigation, tissue sampling for culture/biopsy, and polyethylene exchange
- Magnetic resonance arthrogram (MRA) with intra-articular contrast and diagnostic steroid injection
Corrent answer: 2
The clinical scenario and radiographs are consistent with a patient who is experiencing complex regional pain syndrome following total knee arthroplasty (TKA).
Complex regional pain syndrome, which was previously known as reflex sympathetic dystrophy, is characterized by intense burning pain, stiffness, swelling, and discoloration of the legs, feet, arms and hand (most common). Current treatment modalities are multi- modal and include GABA agonists, alpha-blockers, beta-blockers, physical therapy, occupational therapy, graded motor imagery, tactile discrimination treatments, sympathectomy, local anesthetics, and even spinal cord stimulators.
Mont et al. reported limited success in 27 patients who had surgical exploration of radiographically normal knees following TKA with unexplained pain. Outcomes were especially poor in patients who had achieved adequate range of motion and continued to have pain prior to surgical exploration. Patients with decreased range of motion who achieved improvement in motion postoperatively also demonstrated great relief of pain.
Figure A and B are AP and lateral radiographs of a well-fixed total knee
arthroplasty.
Incorrect Answers:
Answer 1: An MRI of the lumbar spine is not indicated in this patient as their symptoms are suggestive of complex regional pain syndrome, not an L3 radiculopathy.
Answer 3: Surgical exploration of radiographically normal knees following TKA for patients with unexplained pain has been found to have limited success. Answer 4: Surgical irrigation and debridement is not indicated in this setting as the patient is not presenting with symptoms suggestive of infection.
Answer 5: An MRA with intra-articular contrast and diagnostic steroid is not indicated in the setting of complex regional pain syndrome.
-
Which system of lacunar networks is used among osteocytes to communicate?
- Volkmann canals
- Cement lines
- Secondary messenger systems
- Canaliculi
- Haversian canals
Corrent answer: 4
Canaliculi are a system within the lacunar network used by osteocytes to communicate with each other.
Osteocytes have numerous cell processes (filopodia) that project through the canaliculi and connect to each other via gap junctions (see Illustrations B and C). Through this network osteocytes establish contact and communication with adjacent osteocytes for exchange of nutrients and metabolic waste. They are oriented in a radial fashion around the central Haversian canal.
Knothe Tate et al described the changes occuring in different pathologies. In normal bone, osteocyte connectivity is high and processes are oriented in the direction of blood supply. In osteoporotic bone, there is decreased osteocyte connectivity and orientation and tortuous cell processes. In osteoarthritic bone, there is decreased osteocyte viability and connectivity, but preserved
orientation. In osteomalacic bone, there is preserved viability and connectivity, but tortuous and chaotic cell processes.
Illustration A shows the structure of compact and spongy bone. Illustration B shows gap junctions between cells. Illustration C shows secondary messengers crossing gap junctions.
Incorrect Answers
- Volkmann canals run perpendicular to Haversian canals within osteons. They connect Haversian canals to each other and to the periosteum.
- Cement lines mark the boundary of each osteon.
- Secondary messenger systems use small metabolites such as cAMP, IP3 and cADP ribose. These secondary messengers permeate across connexin gap junctions to allow
cell-cell communication (Illustrations B and C).
5. Haversian canals are found in the center of an osteon in compact bone. They contain blood vessels and nerves.
-
A prosthetic foot which incorporates a multi-axis articulated foot assembly is recommended for which of the following amputees?
- Low functioning diabetic who needs to transfer bed to chair
- Long distance runner with below knee amputation
- Elderly male with above knee amputation
- Below knee amputee who needs to regularly walk on uneven ground
- 10-year-old male with above knee amputation from osteosarcoma
Corrent answer: 4
The multi-axis articulated foot assembly is the best prosthetic for ambulation over uneven ground, and functions best for below knee amputations. Low functioning patients who need a prosthesis for transfers would probably benefit from a solid ankle, cushioned heel prosthesis. Runners and athletes often require high end carbon fiber prostheses. Patients with above knee
amputations have several options to optimize ambulation including solid, energy storing, and multi-axial prothesis.
Aaron et al. reviews the important considerations for patient specific prostheses and new developments on the horizon to maximize ambulation in prosthesis users.
-
Mesenchymal stem cells have the capacity to differentiate into all the following cell types EXCEPT?
- Osteoclasts
- Chondrocytes
- Adipocytes
- Osteoblasts
- Fibroblasts
Corrent answer: 1
Mesenchymal stem cells are multipotent stem cells which retain the ability to self-renew and to form cells of the mesenchymal lineage. They can therefore form muscle, fat, tendon (made from fibroblasts), bone, cartilage, and the marrow stromal cells. Mesenchymal stem cells do not however form osteoclasts, which are formed from the monocyte lineage - from hematopoietic stem cells.
Caterson et al review the use of mesenchymal stem cells in tissue engineering and regeneration of musculoskeletal tissue.
-
A 60 year-old male was brought into the operating room for total hip replacement. Before making the incision, what precautionary procedure must be performed by the entire staff to minimize surgical error?
- Mark the word "No" on the nonoperative extremities
- Use intraoperative fluoroscopic imaging
- Perform "timeout"
- Have blood products ready in the operating room
- Use the newest prosthesis
Corrent answer: 3
Surgical "timeout" is now part of the standard procedure which must be performed before starting surgery to minimize surgical error such as wrong site surgery.
-
Which of the following statements regarding articular cartilage is TRUE?
- Cartilage is an isotropic material
- Most of the water in articular cartilage exists in the deep layer next to the calcified cartilage
- Cartilage only heals if the injury does not pass through the tidemark
- Calcified cartilage is the only place that type IV collagen is found
- Cartilage exhibits stress shielding of the solid matrix components
Corrent answer: 5
Cartilage exhibits significant stress shielding of the solid matrix components due to its high water content, the non-compressibility of water, and the structural organization of collagen and proteoglycans. Cartilage is composed of a permeable porous matrix and 65 to 80% of the total weight of articular cartilage is made up of water. A pressure gradient causes the water to flow through the porous-permeable solid matrix. Significant flow of fluid through the solid matrix requires high hydrodynamic pressures because of the low
permeability of the solid matrix.
The other answers are incorrect because cartilage is anisotropic, most of the water is located in the superficial layers, it only heals if the injury does pass through the tidemark, type X collagen is found in calcified cartilage and is thought to be involved in mineralization. Type IV collagen is found in the basal lamina.
-
A 38-year-old patient presents 6 months after intramedullary nailing of a distal third tibia fracture with symptoms consistent with complex regional pain syndrome. During the early stage of the disease he was treated with intermittent splinting, elevation and massage, contrast baths, and transcutaneous electrical nerve stimulation. Despite these modalities, he continues to have severe and debilitating symptoms. Which of the following treatment options is indicated as a second line of treatment?
- Long leg cast immobilization for 3 months
- Walking boot with non weight bearing for three months
- Exchange nailing to stimulate healing response to the limb
- Epidural spinal cord stimulator
- Surgical sympathectomy of the affected limb
Corrent answer: 5
Complex regional pain syndrome is a chronic progressive disease of unknown etiology characterized by pain, swelling and skin changes. If nonoperative modalities fail, a surgical sympathectomy of the affected limb is indicated.
The first line of treatment is physical therapy including intermittent splinting, elevation and massage, contrast baths, and transcutaneous electrical nerve stimulation. Aggressive passive range-of-motion exercises should be avoided. If nonoperative modalites fail and symptoms remain severe, a surgical sympathectomy of the affected limb is indicated.
Keys to successful treatment include early clinical suspicion and treatment. Late CRPS is highly refractory to treatment and often results in permanent disability.
Two forms of Complex regional pain syndrome exist: 1) Reflex sympathetic dystrophy
- which does not demonstrate nerve lesions, and 2) Causalgia - which is associated with damage to peripheral nerves. Diagnostic criteria include:
Major criteria: intense and prolonged pain, swelling, stiffness, and discoloration (vasomotor disturbances).
Minor criteria: trophic changes, osseous demineralization, temperature changes, and palmar fibromatosis.
Tran et al present their systemic review of 41 RCTs of the research regarding treatment of CRPS. Their data suggest that only bisphosphonates offer clear medicinal benefits in the treatment of CRPS. Evidence regarding a beneficial effect of lumbar sympathetic blocks, gabapentin, and physical therapy is lacking. As such, these authors advocate for further study thru well-designed RCTs to better evaluate appropriate and effective treatment strategies.
-
You are planning an intramedullary nail to treat a geriatric patient with a peritrochanteric femur fracture. Which of the following preoperative considerations is correct regarding your implant?
- The radius of curvature of an intramedullary nail is generally greater than the radius of curvature of the femur
- Closed section nails have less stiffness than slotted nails
- The medial/lateral nail starting point relative to the greater trochanter does not affect varus/valgus position in the fracture
- The bending stiffness of your nail is proportional to the second power of the radius
- Intramedullary nails allow for mostly direct intramembranous bone healing
Corrent answer: 1
The radius of curvature of an intramedullary nail is generally greater than the radius of curvature of the femur, which is why anterior distal femur
penetration is a known complication of intramedullary nailing procedures.
Egol et al. evaluated the radius of curvature of 948 femurs (474 matched pairs) and compared those data with current intramedullary nails. He found the average femoral anterior radius of curvature was 120 cm (± 36 cm)
whereas the radius of curvature of the intramedullary nails ranged from 186 to 300 cm.
The other answers are incorrect because closed section nails have more stiffness than slotted nails. The starting position on the greater trochanter greatly affects the post- operative varus/valgus of the fracture. Intramedullary nails allow for mostly indirect enchondral bone healing due to relative motion at the fracture site.
-
All of the following antibiotics function by interfering with protein synthesis by inhibiting ribosomes EXCEPT
- gentamicin
- tobramycin
- vancomycin
- erythromycin
- linezolid
Corrent answer: 3
Gentamicin and tobramycin are aminoglycosides that function by inhibition of bacterial protein synthesis via irreversible binding to ribosomal subunits. Erythromycin functions by binding to the 50s subunit of the bacterial 70s rRNA complex and thereby inhibits protein synthesis. Linezolid binds to the 23s portion of the ribosomal subunit and inhibits protein synthesis. In contrast, Vancomycin acts by inhibiting proper cell wall synthesis and does not inhibit the ribosome.
-
The bending rigidity of the implant shown in Figure A is proportional to what power of the measured radius of the implant?
1. 2
2. 3
3. 4
4. 5
5. 6
Corrent answer: 3
The bending rigidity of a solid cylindrical pin is related to the fourth power of the pin’s radius.
A hollow, cannulated intramedullary nail has a bending rigidity related to the
3rd power. The rigidity of a fracture plate is proportional to the plate thickness to the third power. Thus, doubling the fracture plate thickness increases its bending stiffness 8 times.
The bending rigidity of an external fixator pin is proportional to the fourth power of the pin diameter. The bending stiffness of each pin is proportional to the third power of the bone- rod distance. However, the most important factor external fixator stability is for the the fracture ends to come into contact with each other.
-
Which of the following biochemical changes are common to both aging cartilage and osteoarthritic (OA) cartilage?
- Increased water content
- Decreased collagen content and decreased modulus of elasticity
- Decreased proteoglycan content
- Increased chondroitin sulfate concentration
- Decreased keratin sulfate concentration
Corrent answer: 3
Both aging cartilage and osteoarthritic cartilage share the common change of decreased proteoglycan content as shown in Illustration A. The review article by Martin et al states that OA is not an inevitable consequence of aging but rather, aging increases the risk of OA because of a decrease in the ability of chondrocytes to maintain and repair the tissue. Increased water content and chondroitin sulfate concentration are seen with osteoarthritis but not aging. Decreased collagen content, modulus of elasticity, and keratin sulfate concentration are also seen with osteoarthritis but not aging.
-
The active form of vitamin-D (calcitriol) is produced by the enzyme 1-alpha- hydroxylase. What hormone activates this enzyme?
- thyroid stimulating hormone (TSH)
- parathyroid hormone (PTH)
- estrogen
- progesterone
- testosterone
Corrent answer: 2
The precursor to calcitriol is stored in the skin where UV exposure converts it to previtamin D3. Then, it is bound to vitamin-D binding protein (DBP) where
it is carried to the liver and metabolized to 25-hydroxyvitamin D. When calcium is low, parathyroid hormone (PTH) levels become elevated which activates 1-alpha- hydroxylase which converts it to the active Vitamin D (calcitriol). Its most potent effect is to enhance intestinal calcium absorption through DNA regulation. Illustration A shows a diagram of Vitamin D metabolism.
-
A hospital has recently moved from a paper to a computerized physician order entry (CPOE) system. Compared to the previous safety auditing reports, the hospital can now expect which of the following?
- A decrease in nephrotoxic drug dosing
- No improvement in prescribing behaviors
- An increase in patient satisfaction following use of patient controlled analgesia (PCA)
- An increase in sentinel events
- No effect on physician prescribing errors
Corrent answer: 1
Medication ordering error rates are the most common preventable medication errors. CPOE has improved safety in drug ordering. CPOE has resulted in a decrease in nephrotoxic drug dosing. Kaushal et al performed a systematic review of safety resulting from use of CPOEs. They concluded that most
studies were underpowered and used institutional based systems. However, there was a substantial decrease in serious medication error rates, antibiotic- associated medication errors, adverse drug events, improvements in prescribing behaviors and nephrotoxic drug dosing. Teich et al evaluated CPOEs and determined that computer prompts for dosing and frequency for medications led to fewer provider errors and increased patient safety.
-
Type X collagen expression by hypertrophic chondrocytes is characteristic of which of the following aspects of fracture healing?
- Inflammation
- Granulation tissue formation
- Cartilage callus formation and calcification
- Bone deposition
- Bone remodeling
Corrent answer: 3
There are three phases to fracture healing: 1) Reactive, 2) Reparative, and 3) Remodeling. The reactive phase is characterized by inflammation and granulation tissue formation. The reparative phase is marked by cartilage callus formation and bone deposition. Finally, the bone deposited during the
reparative phase is remodeled during the remodeling phase. Type X collagen is a homotrimeric collagen found in hypertrophic cartilage expressed during the cartilage callus calcification phase of fracture healing. After fracture, inflammation and clot formation occurs where type I and II collagen are found. Type III collagen is expressed by fibroblasts and type V is found in areas of fibrous tissue formation. In the soft tissue callus/chondroid phase, types II and IX predominate, with type II being deposited in areas of mature cartilage production and type IX stabilizing the fibrils of type II. In the callus
calcification phase type X collagen is expressed by proliferating chondrocytes as the extracellular matrix undergoes calcification. In the osteogenic bone deposition phase, there is a progressive shift from primary to secondary spongiosa and type I collagen predominates.
-
Which of the following statements is true regarding the ethical management of orthopaedic implants?
- A surgeon who receives monetary gain from a device he/she has developed may not use this device on his/her own patients.
- Only studies with positive results should be published. Studies with negative findings should be dismissed.
- It is acceptable to rely on the word of the industry sales representative when
choosing an orthopaedic implant during preoperative planning.
- It is the responsibility of the surgeon to be aware of the implant's clinical track record.
- Only implants with support from peer-reviewed clinical literature may be used in the operating room.
Corrent answer: 4
General representations about the safety or effectiveness of specific orthopaedic services should not be misleading. Such representations may cause a layperson to lack
appreciation for the nature of any risks or adverse effects associated with the orthopaedic procedure, even if the likelihood that adverse effects may occur is low. Representations about the safety or effectiveness of orthopaedic services should be substantiated with sound scientific support, such as peer reviewed publications in medical literature or other authoritative sources of scientific information. Such claims should not contradict or be inconsistent with conclusions reached by authoritative federal agencies unless such a contradiction or inconsistency can be substantiated with sound scientific evidence.
The attached link to the AAOS has several thought provoking and instructional case studies in ethics that orthopaedic surgeons frequently encounter. http://orthoportal.aaos.org/estudy= /resident_ethics.asp=x#tab2
-
The paired Student's t-test is most appropriately used for which of the following?
- Determining if the medians are equal in two observed samples
- Discerning differences among a group of more than two means
- Illustrating an actuarial method of survival rates
- Refining a correlation coefficient among outlying observations
- Evaluating the difference between two observed means in matched groups
Corrent answer: 5
The paired Student’s t-test is based upon a probability distribution used to test a hypothesis involving numerical data. This method is used to compare the difference between two observed means. When the observations are not normally
distributed, the Wilcoxon rank-sum test is more powerful than the t- test in detecting an actual difference between paired samples. It is appropriate for small samples that are not normally distributed.
-
Which of the following mediators reduces bone resorption?
- Calcitonin
- RANK ligand
- Interleukin-6
- Parathyroid Hormone
- Dexamethasone
Corrent answer: 1
Calcitonin is a hormone secreted by the parafollicular cells of the thyroid gland to slow bone resorption, reduce calcium resorption in the tubules of the
kidney, and reduce serum calcium. It opposes the effects of parathyroid hormone. Calcitonin secretion is upregulated by hypercalcemia, gastrin, and pentagastrin.
The articles by Zaidi et al and Athanasou et al review the cellular mechanisms behind bone resorption in a review of the basic sciences. They focus on key molecular mechanisms underlying the formation, function, and survival of osteoclasts. These include key signals mediating osteoclast differentiation, including RANK/RANK-L, and key specializations of the osteoclast, including HCl secretion driven by H+-ATPase and the secretion of collagenolytic enzymes including cathepsin K and matrix metalloproteinases (MMPs).
Incorrect Answers:
- RANK Ligand is involved in increasing bone resorption.
- Interleukin-6 is involved in increasing bone resorption.
- Parathyroid hormone increases bone resorption in the process of increasing serum calcium. It opposes calcitonin.
- Dexamethasone is a steroid. Long term use can result in osteoporosis.
-
A 20-year-old male sustains bilateral humeral shaft fractures and bilateral femoral shaft fractures from a motor vehicle collision. While awaiting surgery, the patient suddenly becomes tachycardic,
hypoxemic and experiences mental status changes; physical exam demonstrates multiple petechiae in his bilateral axilla. CT angiography is ordered and is negative for pulmonary embolus. What is the most likely diagnosis?
- hypovolemic shock
- stroke
- fat embolism syndrome
- sundown syndrome
- narcotic overdose
Corrent answer: 3
The clinical presentation is consistent with fat emboli syndrome. Fat embolism syndrome results when embolic marrow fat macroglobules damage small vessel perfusion leading to endothelial damage in pulmonary capillary beds leading to respiratory failure and an ARDS-like picture. Patients following long bone fractures are particularly susceptible to fat emboli syndrome. Fat emboli syndrome occurs in only 3-4% of patients with long bone
fractures, despite nearly 90% of patients with long bone fractures having some evidence of fat emboli. The major clinical features of FES include hypoxia, pulmonary edema, central nervous system depression, and axillary or subconjunctival petechiae.
Levy et al review fat emboli syndrome in the referenced article, and note that supportive measures, including positive pressure ventilation and effective fluid management, are important. They also recommend expedited fixation of long bone fractures to minimize ongoing embolic phenomenon.
Gurd et al. developed a criteria system to diagnose fat emboli syndrome based on the presence of major and minor criteria (see topic review for list of criteria). Key to treatment is maintaining perfusion to vital organs. Treatment is largely supportive.
-
A research study is initially designed as a high-quality randomized control trial. The study seeks to demonstrate a different clinical outcome with treatment A compared to treatment B for a specific disease. However, at the completion of the study the investigators are only able to obtain 70% follow-up of patients. What level of evidence should be assigned to this study?
1. 1
2. 2
3. 3
4. 4
5. 5
Corrent answer: 2
A randomized clinical trial (RCT) is a study design that is appropriate to demonstrate statistically that a clinical outcome in one treatment group is different from that in another treatment group. A properly designed RCT that meets certain criteria is considered Level 1 evidence.
AAOS Comprehensive Orthopaedic Review and the article by Wright et al states that randomized control trials that have "less than 80% follow-up, no blinding, or improper randomization" are lesser-quality studies and qualify as Level 2 evidence. This is also the criteria used by JBJS-Am for assigning levels of evidence for RCT's. The RCT in this question has a 70% follow up and therefore would be considered Level 2 evidence. Level 2 evidence also includes prospective cohort studies whereby patient groups are separated non- randomly by exposure or treatment, with exposure occurring after the
initiation of the study. Level 3 evidence includes retrospective cohort studies and case- control studies whereby patient groups are separated by the current presence or absence of disease and examined for the prior exposure of interest. Level 4 evidence includes a case series of multiple patients with the same treatment, but no control group or comparison group. Level 5 evidence includes a case report, expert opinion, or anecdotal evidence.
-
After being given explicit instructions to sign their operative limb before surgery to prevent wrong-side surgery, what percentage of Worker's Compensation patients were non-compliant with these instructions?
1. 0%
2. 10%
3. 50%
4. 70%
5. 100%
Corrent answer: 4
DiGiovanni et al conducted a prospective study of 100 patients to examine compliance in marking the incorrect side of surgery before arriving at the hospital. 59% were compliant, 4% were partially compliant, and 37% were non- compliant. 70% of Worker's Compensation patients were non-compliant with
preoperative instructions. This study suggests that patients expect the system to "take care of everything" despite solicitation of their active participation to avoid such adverse events.
-
During fracture healing, granulation tissue tolerates the greatest strain before failure so that mature bone can eventually bridge the fracture gap during healing. What is the definition of strain?
- Amount of force an object can withstand until plastic deformation is lost
- Change in length / original length of an object due to an external force
- Relationship of stiffness to time-dependent loading
- Force intensity / volume
- Force intensity / cross sectional area
Corrent answer: 2
Strain is defined as the change in length/original length (L) and is created by a deformation of a material from an applied force.
The mechanical environment at the fracture site has a major influence on fracture healing. Granulation tissue can withstand higher strain, which stabilizes the mechanical environment and forms a scaffold on which cartilage and bone eventually form; this occurs after strain decreases incrementally. Optimal healing, however, depends on duration, rate, timing and type of mechanical influence. Bone is formed by osteoblasts that are adapted to the very low strains of over 1% change in length. Osteoblast synthesis and proliferation is stimulated at uniaxial strain of between 0.3% and 2.8%. It is known that limited inter-fragmentary movement of 0.2 mm to 1 mm is optimal for fracture
healing, resulting in promotion of callus and increase in rigidity. Excessive movement, on the other hand, prolongs fracture healing. Researchers have identified that tissue strain of 2% is suitable for primary
bone healing and secondary bone healing takes place at tissue strain of 2-
10%. Strain of 10-100% results in fibrous tissue formation and 100% strain to non-union. This is known as Perren's theory.
Stokes published a review article on the effects of stress on bone healing and growth, and notes the importance of the 'Hueter-Volkmann Law' (growth is retarded by increased mechanical compression, and accelerated by reduced loading in comparison with normal values) in bone growth. Stokes also notes that sustained compression of physiological magnitude inhibits growth by 40%
or more, while distraction increases growth rate by a much smaller amount.
Illustration A shows an example of a stress-strain curve, with several key definitions labeled on the diagram.
Incorrect Answers:
Answer 1: This is the elastic limit or yield point.
Answer 3: The time-dependent deformations caused by creep and shrinkage have an Answer effect on the stiffness or modulus of an object.
Answer 4: This is the definition of force density. Answer 5: This is the definition of stress.
-
Which of the following materials is most susceptible to galvanic corrosion?
- Titanium
- Zirconia
- Polyethylene
- Cobalt-chromium
- Alumina
Corrent answer: 4
Of the materials listed, cobalt-chromium is the only material that is most susceptible to galvanic corrosion. Galvanic corrosion is defined as intense localized electrochemical attack between two metal components exposed to
corrosive environments.
Incorrect Answers:
Answer 1: Titanium has less galvanic corrosion than cobalt-chromium alloys because it chemically protects itself by a reaction called self-passivation, which is the formation of a protective surface oxide.
Answer 2 & 5: Zirconia and alumina are both ceramics, and are immune to metallic galvanic corrosion.
Answer 3: Polyethylene is a plastic polymer which is also immune to metallic galvanic corrosion.
-
The sclerotome of the somites develops into what mature anatomical structure?
- Autonomic nervous system
- Dorsal muscles
- Ventral muscles
- Appendicular skeleton
- Axial skeleton
Corrent answer: 5
The somites are mesodermal in origin; somites eventually form dermis (dermatome), skeletal muscle (myotome), sclerotome or cartilage, tendons, and endothelial cells. The sclerotome of the somites develops into the axial skeleton. The neural crest forms the peripheral nervous system, the lateral plate mesoderm becomes the dermis, and the dorsal myotome (epimere) becomes the dorsal muscles.
Illustration A depicts the sclerotome of the somites development into the vertebra. The nucleus pulposus forms from the notochord and the annulus fibrosus forms from the sclerotome.
-
All of the following antibiotics function by interfering with cell wall synthesis EXCEPT
- Cefazolin
- Penicillin G
- Vancomycin
- Imipenem
- Gentamicin
Corrent answer: 5
Cephalosporins (cefazolin), penicillins, vancomycin, and imipenem function by interfering with cell wall synthesis. Gentamicin, an aminoglycoside, functions by inhibiting ribosomes and protein synthesis and does not affect cell wall synthesis.
The reference by Mader et al. is an instructional course lecture that reviews
the different mechanisms of antibiotics and their indications in musculoskeletal infections. Illustration A is a table showing common mechanisms of antibiotics.
-
What is the most common radiographic finding in reflex sympathetic dystrophy (RSD) or complex regional pain syndrome of the knee?
- patella baja
- patella alta
- patella osteopenia
- generalized osteopenia
- supracondylar stress fracture
Corrent answer: 3
Reflex Sympathetic Dystrophy (RSD) of the knee is different than that of the upper extremity. Pain out of proportion to the initial injury is the hallmark symptom. Other features include vasomotor disturbances, delayed functional recovery and various associated trophic changes. The JAAOS article by Cooper et al states that patellar osteopenia "is the most common radiographic finding". However, they go on to state that the most reliable diagnostic test is symptom relief after sympathetic blockade. The JBJS article by Cooper et al treated 14 patients with RSD of the knee with epidural blocks for 4 days. Eleven patients had complete resolution of their symptoms, and pain that was out of proportion to the severity of the injury was the most consistent finding. Katz et al reviewed 36 patients with RSD primarily affecting the knee. They found that injuries or operation about the patellofemoral joint triggered its onset in 64% of patients.
-
Tumor necrosis factor receptor 1 is involved in which of the following cellular events?
- Replication
- Agenesis
- Apoptosis
- Senescence
- Ectopy
Corrent answer: 3
Apoptosis is defined as a sequence of events leading to programmed cell death. A cell undergoing apoptosis goes through a series of signaling events in which the cell disposes of itself in a neat and orderly manner. This is in contrast to cell lysis where the cell is destroyed, releasing its contents (including harmful enzymes) and DNA material which is toxic to neighboring cells.
Ashkenazi et al present a review of apoptosis, including the biochemical mechanism behind the programming. They note that the death receptors Fas and tumor necrosis factor receptor 1 (TNFR1) trigger apoptosis upon engagement by their cognate death ligands.
Illustration A shows a diagram of apoptosis, including the cellular mechanisms.
-
The Stark II regulations limit which of the following activities for physicians who accept Medicare and/or Medicaid insurance payments?
- Direct use of implant manufacturer financial support for non-CME courses
- Use of in-office durable medical equipment for patient care
- Physician office space rental at fair market value prices
- Direct physician employment by a hospital system
- Referral of patients to entities in which the physician has a financial relationship
Corrent answer: 5
On Jan. 3, 2001, the United States Department of Health and Human Services (HHS) released the first phase of its long-anticipated final regulations addressing self-referrals by physicians. These regulations clarify self-referral prohibitions under section 1877 of the Social Security Act, often referred to as the "Stark law," which bar physicians from making referrals to entities in which the physician has a financial relationship, and for which no statutory exception applies. Stark II presently provides for civil money penalties not to exceed
$100,000 for each "arrangement or scheme" that a person knows or should know has a principal purpose to violate the statute. Additionally, the government may withhold payments for prohibited referrals or seek to recoup past payments.
-
Which of the following statements best describes a plasmid?
- An extrachromosomal, circular piece of DNA that replicates independently of host DNA
- An extrachromosomal, linear piece of RNA which replicates independently of host DNA
- A protein which promotes transcription of DNA to RNA
- A gene which leads to cancerous cell transformation
- A gene which suppresses cancerous cell transformation
Corrent answer: 1
A plasmid is an extrachromosomal circular piece of DNA that replicates independently of host DNA.
In nature, plasmids frequently carry genes that may benefit the survival of the organism, such as antibiotic resistance genes. Artificially, plasmids are frequently used as vectors to introduce genes into a cell and change genetic expression. This has potential therapeutic benefits for the treatment of
arthritis and bone healing.
Whalen et al. discuss various methods for transferring therapeutic genes into an arthritic joint. These include gene transfer through a vector inserted directly into a joint, and indirect transfer where synovial cells are isolated and genetically modified using vectors, then transplanted back into the joint.
Lieberman et al. present a review discussing the potential future benefits of gene transfer in bone healing, spine fusion, and articular cartilage repair.
Illustration A shows a visual representation of a plasmid. Note how the plasmid is extrachromosomal and the structure is identical to that of the host DNA. Video V is an educational video that gives an overview on plasmids.
Incorrect Answers:
Answer 2: A plasmid is a small piece of DNA, not RNA. Answer 3: This defines a trancription factor.
Answer 4: This defines an oncogene.
Answer 5: This defines a tumor suppressor gene.
-
An 18-year-old female soccer player sustains a non-contact deceleration injury while making a sharp pivot to strike the ball. She hears a loud pop in her knee, is unable to bear weight initially following the injury, and develops an immediate knee effusion. The structure most likely injured in this athlete exhibits all of the following biomechanical properties EXCEPT:
- Viscoelasticity
- Creep
- Isotropism
- Stress relaxation
- Nonlinear elasticity
Corrent answer: 3
The clinical presentation is consistent for an ACL tear. The ACL has the biomechanical properties of viscoelasticity, creep, stress relaxation, and nonlinear elasticity. It does NOT demonstrate isotropism. Isotropic materials such as metals exhibit the same mechanical properties in all directions.
All ligaments and tendons are anisotropic and exhibit different mechanical properties depending on the direction of the applied load. Ligaments are viscoelastic indicating they exhibit a time-dependent mechanical behavior. Thus, the relationship between stress and strain is not constant but depends on the time of displacement or load. One characteristic of viscoelasticity is creep, whereby there is an increasing deformation under constant load. Viscoelastic materials also exhibit stress relaxation whereby stress will be reduced or will relax under a constant deformation. Ligaments also demonstrate nonlinear elasticity (see illustration paragraph below).
Screen et al. studied viscoelasticity within isolated tendon fascicles. Their results provide further evidence of the complex anisotropic and viscoelastic nature of tendons. They conclude proteoglycans play an important functional role in controlling the viscoelastic behaviour and the mechanisms of strain transfer within tendon.
Illustration A demonstrates, nonlinear elasticity, which is another characteristic of ligaments. The toe region (labeled A in Illustration A) represents "un- crimping" of the crimp in the collagen fibrils. Since it is easier to stretch out
the crimp of the collagen fibrils, this part of the stress strain curve (the "toe region") shows a relatively low stiffness. As the collagen fibrils become uncrimped, the collagen fibril backbone itself is being stretched (labeled B in Illustration A), which gives rise to a stiffer material. As individual fibrils within the ligament or tendon begin to fail damage accumulates, stiffness is reduced and the ligament/tendons begins to fail (labeled C in Illustration A).
-
All of the following are desired goals of improved diversity and cultural competence in orthopaedics EXCEPT:
- Understand cultural differences so that patient-physician relationships are enhanced
- Reduce disparities in healthcare
- Establishment of quotas for minority and female medical students
- Access to orthopaedic care is optimized
- Better serve the nation's diverse population
Corrent answer: 3
There are no plans to create quotas in the residency match selection process. The referenced article by White is a symposium on the need for diversity within orthopaedic surgery.
The AAOS has established goals for diversity in an effort to enhance the patient and physician relationship, eliminate disparities in healthcare, and optimize access to orthopaedic care. The AAOS The Diversity Committee endeavors to significantly improve minority and female access to the orthopaedic profession and more information can be found at the referenced academy website (www3.aaos.org/about/diversity/cul_recr.cfm)
-
What is the most important factor when choosing an optimal lower limb prosthesis for an adult patient?
- Prosthesis cost
- Patient comorbidities
- Patient functional status
- Patient gender
- Patient age
Corrent answer: 3
The most important factor when choosing an optimal lower limb prosthesis for an adult patient is the current and potential functional requirements of the patient.
Pinzur reviewed 90 patients treated with lower extremity amputations for peripheral vascular insufficiency and found the current and potential functional status of the patient were more important than any other factor in creating the best lower limb prosthesis for the patient.
-
A 28-year-old marathon runner has a knee MRI done to evaluate medial sided knee pain. No meniscus tear is identified, but the
articular cartilage of the femoral condyles is noted to be twice as thick as normal values. What is the most likely cause of this finding?
- Osteoarthritis
- Undiagnosed osteopetrosis
- Synovial cytokine sensitization due to excess running
- Extra-articular hydrostatic pressure changes
- Chondrocyte modulation via mechanotransduction
Corrent answer: 5
Chondrocyte modulation via mechanotransduction secondary to excess load is a known cause for alterations in the cellular structure of articular cartilage. Specifically, chondrocyte metabolism responds to both mechanical (mechanical load, hydrostatic pressure change) and chemical stimuli (growth factors, cytokines). It is known that articular cartilage remodels quickly following alterations of mechanical stimuli to the tissue in vivo, and can change its metabolic activities in vitro. As such, generalized thickening of articular cartilage in a marathon runner is most likely secondary to the increased mechanotransduction from repeated load.
Incorrect answers:
- Osteoarthritis - associated with joint space narrowing, osteophytic changes, increased subchondral sclerosis, and subchondral cysts.
- Osteopetrosis - a osteoclast disease, not related to chondrocytes
- Cytokine sensitization - would occur on the chondrocytes, not the synovium
- Intra-articular, not extra-articular, hydrostatic pressure changes would affect chondrocytes.
-
Which of the following medications exerts its influence on the clotting cascade by inhibiting the carboxylation of normal clotting factors?
- Warfarin
- Enoxaparin
- Dalteparin
- Heparin
- Hirudin
Corrent answer: 1
Warfarin (Coumadin) exerts its anticoagulation effect by inhibiting the carboxylation of normal clotting factors. Warfarin is a vitamin K antagonist
that prevents the reductive metabolism of vitamin K epoxide back to its active form, hydroquinone, by inhibiting the enzymes responsible for the reaction. The vitamin K- dependent factors are II, VII, IX, X, proteins C, and S.
The reference by Hyers is a review article discussing the antithrombotic agents that have been used in the last 50 years and also discusses some of the newer ones that have since been developed.
Berry in his review discusses the risk factors, efficacy, and safety of agents used in 2003 after total hip arthroplasty.
Answer 2: Enoxaparin binds to and increases the activity of antithrombin III. By activating antithrombin III, enoxaparin potentiates the inhibition of coagulation factors Xa and IIa.
Answer 3: Dalteparin is a low molecular weight heparin
Answer 4: Heparin binds to the enzyme inhibitor antithrombin III. The activated AT then inactivates thrombin and other proteases involved in blood clotting, most notably factor Xa.
Answer 5: Hirudin (the active component released by leeches), is often considered the most potent inhibitor of thrombin.
-
A 25-year-old healthy male is scheduled to undergo a a nine-level posterior spinal fusion for scoliosis. Administering preoperative recombinant erythropoietin would place the patient at increased risk of developing which of the following complications?
- Acute renal failure
- Increased bleeding time
- Thrombotic event
- Wound complications
- Delayed spinal fusion
Corrent answer: 3
The use of recombinant erythropoietin(EPO) preoperatively for patients undergoing major elective orthopedic surgery has been associated with a higher incidence of deep vein thrombosis (DVT).
Johnson et al present a Level 5 review of recombinant eryrthropoetin (Epoetin alfa). When hypoxia in the body is detected, the kidney is stimulated to produce EPO in the renal cortical interstitial cells. EPO interacts with progenitor stem cells in the bone marrow to increase RBC production. The use of erythropoietin does decrease transfusion rates, but has no effect on renal function, bleeding times, wound complications, or bony healing.
The Level 1 study by Beris et al studied the use of recombinant human erythropoietin as an adjuvant treatment to autologous blood transfusions in elective surgery. They reported a 10% DVT rate with the 300 U/kg recombinant EPO (10%) dosage versus placebo (5% rate).
-
A 4-year-old female is brought by her parents in regard to a right sided limp that improves during the day and has been present for two months. She is found to have a right knee effusion and associated
soft-tissue swelling with no redness or warmth. Lab work reveals negative Rheumatoid factor, a positive low titer ANA and a normal WBC. Radiographs are normal for her age. What additional work up does she need?
- Skeletal survey
- MRI of the pelvis
- Clotting factor levels
- Ophthalmology evaluation
- Bone scan
Corrent answer: 4
This patient has a history and physical findings consistent with juvenile idiopathic arthritis (JIA). This type of JIA specifically has a high association
with iridocyclitis, particularly in those with positive ANA studies (approximately
20%). Patients with JIA require an ophthalmology consultation for slit lamp examination to evaluate for anterior uveitis, with any type of pupil asymmetry requiring an immediate consultation. Eye involvement can be indolent and lead to blindness if not promptly identified. Due to early treatment of the uveitis, blindness has become a rare complication.
The Sherry article provides an overview of new treatment methods including intraarticular joint injections of methotrexate and etanercept, which have produced giant leaps in the treatment of the associated joint inflammation and resultant destruction.
-
A 25-year-old male sustains a transverse humeral shaft fracture and undergoes open reduction and internal fixation with rigid compression plating. What kind of bone healing would be expected with this type of fracture fixation?
- Primary bone healing through haversian remodeling
- Secondary healing through callus formation
- Primary healing through callus formation
- Endochondral ossification
- Secondary healing through osteonal cutting cones
Corrent answer: 1
Fractures and osteotomies that are stabilized with rigid compression plating undergo primary bone healing, also known as haversian remodeling. Absolute stability constructs, such as a compression plate, allow for bone healing without visible callus formation.
Healing occurs via extension of clusters of osteoclasts (known as osteonal cutting cones) across the fracture site, along with osteoblasts depositing new bone and blood vessels to re-establish the haversian system.
Seconday bone healing occurs when fractures heal through callus formation. Relative stability constructs, such as an intramedullary nail, allow for some motion at the fracture site which leads to healing through a cartilage scaffold (endochondral ossification).
Illustration A demonstrates a transverse fracture stabilized with a compression plate. Illustration B shows a femoral shaft fracture that has healed through callus formation.
Incorrect Answers:
Answer 2: Secondary healing through callus formation occurs when fractures heal with some motion at the fracture site; for example intramedullary nailing of a diaphyseal femur fracture
Answer3: Primary healing is defined by the absence of visible callus
Answer 4: Endochondral ossification is bone generation or healing through a cartilage scaffold
Answer 5: Secondary healing does not occur through osteonal cutting cones as there not enough stability at the fracture site
-
When analysing complex geometric form and material property distributions, the structure of interest may be divided up into numerous connected subregions or elements within which approximate functions are used to represent the unknown quantity. What is the name for this technique?
- Breakdown synthesis
- Finite element method
- Algebraic conclusion
- Differential equations
- Isogeometric analysis
Corrent answer: 2
To solve a problem with complex geometric form and material property distributions, the finite element approach is used to break the problem up into smaller “finite elements” with simple geometric form. Usually triangular or quadrilateral elements are used. A computer program is written to balance the forces and moments acting on each element, and match these forces and moments with those of its neighboring elements. For large structures with a large number of elements, the computer must solve thousands of algebraic equations to make sure all the forces are balanced in the interior of the body and at the surface where the forces are applied. In orthopedics, stress analysis of the cement fixation of implants to bone is frequently carried out using finite element analysis.
-
A load-elongation curve for a tendon is shown in Figure A. Which of the following statements accurately describes the region labeled "X"?
- The failure region which has crimped tendon fibers
- The linear region which has parallel oriented tendon fibers
- The linear region which has crimped tendon fibers
- The toe region which has parallel oriented tendon fibers
- The toe region which has crimped tendon fibers
Corrent answer: 5
Region "X" in the illustration is the toe region of the load-elongation curve. This region represents the initial elongation during which a small amount of tension causes crimped, randomly arranged fibrils to become aligned parallel along the direction of loading.
Magnusson et al looked at the properties of tendon in relation to muscular activity and training. Collagen composition of tendon is organized in a very hierarchical manner along parallel bundles. Tendon collagen bundles have a more parallel orientation along the long axis than ligaments, making their toe region smaller. Illustration A shows all the regions of the load-elongation curve.
-
Which of the following molecules binds to the surface of hydroxyapatite crystals and prevents protein prenylation?
- Calcitonin
- Parathyroid Hormone
- Raloxifene
- Calcium
- Alendronate
Corrent answer: 5
Bisphophonates accumulate in high concentration in bones due to their binding affinity to hydroxyapatite crystals.
There are two types of bisphosphonates with different mechanisms, although both classes ultimately inhibit osteoclast resorption of bone. Nitrogen containing bisphosphonates (alendronate/Fosamax, pamidronate/Aredia, risedronate/Actonel) prevent protein prenylation by inhibiting farnesyl diphosphate synthase, an enzyme in the mevalonate (cholesterol) pathway.
The non-nitrogenous bisphosphonates (etidronate/Didronel, clodronate, tiludronate) are metabolised in the cell to compounds that replace the terminal pyrophosphate moiety of ATP, forming a nonfunctional molecule that competes with adenosine triphosphate (ATP) in the cellular energy metabolism. Due to this disruption in metabolism, the osteoclast initiates apoptosis and dies, leading to an overall decrease in the breakdown of bone.
-
Which of the following foot radiographs is most consistent with the diagnosis of gout?
- A
- B
- C
- D
- E
Corrent answer: 2
Figure B is most consistent with a diagnosis of gout.
Gout results from deposition of the monosodium urate crystal. It affects the lower limb, resulting in arthritis of the great toe (podagra). On radiographic evaluation, periarticular erosions in the setting of tophaceous formations may be seen.
Egan et al. describe the characteristic radiographic findings of gout in the foot. This includes asymmetric polyarthropathy, well-defined erosions with sclerotic margins, overhanging bony edges and tophaceous formations.
Figure B shows an AP radiograph of a foot affected by gout. Note the periarticular erosions, soft tissue calcifications (tophi), overhanging bony edges and asymmetric joint wear.
Incorrect Answers:
Answer 1: Figure A is consistent with psoriatic arthritis. Notice the bilateral involvement and the pencil-in-cup deformity seen in the great toe.
Answer 3: Figure C is consistent with Freiberg’s infarction. Note the flattening of the second metatarsal head in addition to joint sclerosis
Answer 4: Figure D is consistent with Charcot arthropathy. Note the involvement of the hindfoot. There is fragmentation and severe joint space narrowing
Answer 5: Figure E is consistent with rheumatoid arthritis. Note the loss of asphericity of the 1st metatarsal head, with concomitant dislocations of the
2nd and 3rd MTP joints
-
Which of the following sarcomas is correctly paired with its most common translocation?
- Alveolar rhabdomyosarcoma: t(9;22)
- Synovial sarcoma: t(11;22)
- Ewing's sarcoma: t(12;16)
- Myxoid liposarcoma: t(X;18)
- Clear cell sarcoma t(12;22)
Corrent answer: 5
Chromosomal translocations are characteristically associated with several sub- types of soft tissue sarcomas. The most common clear cell sarcoma translocation is t(12:22). A histologic example is found in illustration A.
Many sarcomas have distinct translocations which can help identify them via cytogenetic testing. The most common are alveolar rhabdomyosarcoma:t(2;13), synovial sarcoma:t(X;18), Ewing’s sarcoma: t(11,22), myxoid liposarcoma:t(12;16), and chondrosarcoma:t(9;22). Histologic examples are in Illustrations B through F, respectively. Osteosarcoma does not have a characteristic translocation.
In a review article, Rabbits described many fusion proteins resulting from chromosomal translocations. As many are nuclear proteins, future molecular
based therapies are being developed to target steps from oncogene transcription to RNA translation. Solomon et al reviewed chromosome aberrations in rare and common tumors. A broader understanding of chromosomal abnormalities and fusion proteins will aid gene-targeted diagnosis and therapies.
-
A 65-year-old female undergoes a total knee arthroplasty. In addition to chemoprophylaxis for deep vein thrombosis (DVT) prevention she is given pneumatic compression devices. Which of the following is associated with pneumatic compression devices?
- Increased endothelial fibrinogenesis
- Decreased bleeding times
- Increased endothelial injury
- Increased venous compliance
- Increased venous blood flow
Corrent answer: 5
External pneumatic compression devices have been shown to prevent the formation of DVTs. Modern devices evacuate blood from lower-extremity vessels in an automated fashion. Pneumatic compression may exert its protective effect against thrombus formation in part by increasing venous blood flow. Pneumatic compression devices do
not decrease bleeding time or cause endothelial injury. Pneumatic compression devices enhance endothelial derived fibrinolysis and decrease venous compliance.
Rogers et al present practice management guidelines for DVT prophylaxis in trauma patients. They state that the exact mechanism of action of pneumatic compression devices are not fully understood. However, there is good evidence that they increase mean and peak femoral vein velocity and there are a few studies concluding that the fibrinolytic system is activated.
-
Which of the following substances is most osteoinductive?
- Calcium phosphate
- Hydroxyapatite
- Xenograft collagen sheet
- Cancellous allograft
- Cancellous autograft
Corrent answer: 5
An ideal bone-graft substitute must provide scaffolding for osteoconduction as well as progenitor cells and growth factors for osteoinduction. Furthermore, the bone graft must be able to integrate with the host. Autogenous bone graft contains osteoblasts, endosteal osteoprogenitor cells capable of synthesizing new bone, and a structural matrix that acts as a scaffold, making it the gold standard for bone grafting. BMP-2 is a commonly utilized adjunct for grafting, and is inherently osteoinductive.
The referenced article by Buckwalter et al is a review on the biology of bone grafting which nicely defines the various osteoinductive and osteoconductive properties of the various bone graft options.
-
Regarding skeletal muscles, which of the following is true?
- Force generated is most dependent on muscle length
- Force generated is most dependent on muscle fiber type
- Type I muscle is comprised of fast twitch fibrils
- Duration and speed of contraction are most dependent on cross-sectional area
- Duration and speed of contraction are most dependent on muscle fiber type
Corrent answer: 5
The duration and speed of contraction is most dependent on the muscle fiber type. The force generated by the muscle is most dependent on the cross- sectional area of the muscle.
Fiber types have less to do with the force of contraction and more to do with the duration and speed of contraction. The cross-sectional area of a muscle determines to a great extent the force generated by the muscle and is controlled by the number of myofibrils that contract. Muscle length affects contraction force through the Blix curve. The morphology of a muscle can affect the cross-sectional area by varying the angle of the fibers in relation to the force vector.
Incorrect Answers:
- Force generated is most dependent on muscle cross-sectional area.
- Force generated is most dependent on muscle cross-sectional area.
- Type I is "slow twitch" and Type II is "fast twitch"
- Duration and speed of contraction is most dependent on fiber type.
-
The nonunion as seen in Figure A will most likely unite by what intervention?
- Increased mechanical stability
- Decreased mechanical stability
- Increased biology at the fracture site
- Decreased biology at the fracture site
- Antibiotics and resection of pseudoarthrosis
Corrent answer: 1
As described in the review by Rodriguez-Merchan and Forriol, hypertrophic nonunions result from motion at the fracture site and generally unite once the mechanical stability is increased. Atrophic and oligotrophic nonunions, while multi-factorial, result from poor biology at the fracture site (poor vascularity, lack of mesenchymal stem cells, bone loss). Treatment of atrophic nonunions then entails takedown of the nonunion and bone grafting (to improve the biology) with stabilization to initiate a healing response.
-
A 53-year-old male laborer presents to his primary care physician with complaints of acute onset of left knee pain. He has had mild episodes of knee pain in the past and is two years status post a left partial medial meniscectomy. He has had mild relief with the use of anti-inflammatories. His past medical history is significant only for
hyperparathyroidism and mild hypertension. He denies any fevers or chills. His exam reveals a moderate knee effusion and diffuse pain and tenderness with palpation and range of motion. Weightbearing radiographs are shown below. The most likely etiology of the patient's knee pain is characterized by which finding?
- Deposits of monosodium urate crystals
- Deposits of calcium pyrophosphate-dihydrate crystals
- Destructive pannus formation
- Empty osteocyte lacunae
- Recurrent hemarthroses
Corrent answer: 2
The patient has chondrocalcinosis, or pseudogout. This disease is characterized by radiographs that show deposits of calcium pyrophosphate- dihydrate crystals in the articular cartilage and menisci. Aspiration typically yields a moderate WBC count (<50,000) and weakly positive birefringent rhomboid-shaped crystals as shown in Illustration A.
Fisseler-Eckhoff and Muller examined over 3,000 menisci from arthroscopic retrieval for evidence of chondrocalcinosis followed by acquisition of clinical data to evaluate for a possible association with prior surgery or trauma. Their results revealed a positive correlation between prior surgery or traumatic knee
injuries and the presence of chondrocalcinosis in 68.6% of patients.
Hough and Webber discussed multiple pathologies affecting the menisci, including chondrocalcinosis. They report on the high association of chondrocalcinosis with the presence of OA within the knee, but conclude that the exact relationship of these two pathologies has yet to be fully elucidated.
Incorrect Answers:
Answer 1: Refers to gout which can display punched out periarticular erosions on radiographs and strongly negative birefringent needle-shaped crystals, as shown in Illustration B.
Answer 3: Characteristic of rheumatoid arthritis, which is associated with symmetric joint space narrowing, osteopenia and periarticular erosions Answer 4: Consistent with osteonecrosis, which is not apparent on the radiograph
Answer 5: Suggests hemophilia. Typical presentation includes a history of bleeding events and radiographs that show a widened intercondylar notch and enlarged femoral condyles
-
A 52-year-old woman falls stepping off the escalator and sustains the wrist fracture shown in Figures A and B. Post-reduction radiographs demonstrate 20 degrees of residual dorsal angulation. The decision is made to proceed with open reduction internal fixation with a volar plate. Which of the following adjuvant interventions has studies in the literature to support an improvement in outcomes?
- Application of a bone stimulator within one week following surgery
- Supplemental percutaneous pin fixation that is removed 4 weeks following surgery
- Immobilization of the wrist in an extension splint or cast for 3 weeks following surgery
- Administration of oral vitamin C beginning the first day after surgery
- Use of an axillary regional block during the surgery
Corrent answer: 4
The radiographs demonstrate a distal radius fracture. Given the residual angulation, open reduction internal fixation is necessary. Supplemental vitamin C has been shown in 2 separate level 1 studies to reduce the incidence of
reflex sympathetic dystrophy. However, it should be noted that subsequent studies have had opposite conclusions.
Cazanueve et al evaluated 195 patients who underwent open reduction internal fixation of a distal radius fracture. The first 100 did not receive vitamin C postoperatively. The next 95 patients received oral vitamin C for 45 days beginning the first day after surgery. The patients who received vitamin C had 5 times less incidence (2% vs. 10%) of reflex sympathetic dystrophy.
-
Salter-Harris type I fractures typically occur through which zone of the physis?
- Resting zone
- Proliferative zone
- Zone of maturation
- Zone of degeneration
- Zone of provisional calcification
Corrent answer: 5
Physeal fractures occur in zone of provisional calcification, which is a part of the hypertrophic zone. The transition from the soft cartilage to to the hard calcified metaphysis puts this zone at risk for fracture.
Illustration A depicts the different zones of the physis and their corresponding diseases.
-
An orthopaedic surgeon is a team physician for a college football team. In which of the following scenarios is it appropriate to discuss a
19-year-old collegiate football player's lumbar spondylosis without additional consent from the player?
- Discussing with the player's mother
- Discussing with the player's treating chiropractor
- Discussing with the player's long-time highschool football coach
- Discussing with a reporter from a local newspaper
- Discussing with the player's fiancee
Corrent answer: 2
Because the player has previously established care for a particular medical condition in the past, per HIPPA rules, it is legal for the team physician to discuss this issue with the treating chiropractor.
The Health Insurance Portability and Accountability Act (HIPAA) Privacy Rule, intended to address potential threats to patient privacy posed by the computerization and standardization of medical records. In most cases, compliance with the Privacy Rule was required as of April 2003.
The privacy rules do not require an individual’s written authorization for certain permitted or required uses and disclosures of the medical records. Patient or parental authorization is not required for disclosures for certain purposes related to treatment, payment, or health care operations. Specifically, HIPAA does not require a covered entity to obtain patient authorization for many of the health care industry’s most fundamental activities such as providing care.
-
An orthopaedic surgeon is closing a total knee replacement incision of a patient with HIV and accidently sustains a needlestick. What is the surgeon's approximate risk of contracting the virus?
1. 0.3%
2. 3%
3. 10%
4. 20%
5. 30%
Corrent answer: 1
The average risk of transmission of human immunodeficiency virus (HIV) to a health care worker after percutaneous exposure to HIV-infected blood has been estimated as 0.3 percent. Cardo et al found the risk of HIV infection increases with a larger volume of blood and a higher titer of HIV in the source patient's blood. Postexposure prophylaxis with NRTIs (nucleoside reverse transcriptase inhibitors) appears to be protective.
-
A 12-year-old boy with sickle cell anemia complains of 24 hours of pain in the right thigh. He denies any traumatic injury. The right leg has tenderness with palpation of the mid-thigh. The patient's temperature is 100.2 degrees F, and ESR is
45. Radiographs of the femur are unremarkable. A radionuclide bone scan demonstrates abnormal uptake in the mid-femur. A radionuclide bone marrow scan demonstrates decreased uptake within the marrow. Which of the following is the best step in management?
- Symptomatic care for his pain with NWB crutches, intravenous hydration, and consultation with hematology
- Steroid injection of the quadriceps
- Two weeks of an oral cephalosporin and follow-up radiographs
- Bone biopsy for culture and intravenous antibiotics
- Surgical debridement, culture, and intravenous antibiotics
Corrent answer: 1
Sickle cell crises resulting in bone infarcts can be difficult to differentiate from acute osteomyelitis with physical exam and plain radiographs alone. This child is undergoing a bone infarct as confirmed by his radionuclide scans.
Skaggs et al reviewed 79 cases of acute extremity pain in sickle cell patients. Radionuclide bone marrow and bone scan was used to differentiate osteomyelitis from bone infarct. Four cases of infection were diagnosed by normal uptake on the bone marrow scan and abnormal uptake on the bone scan. These cases were confirmed osteomyelitis by positive culture. Seventy cases were diagnosed as bone infarct by decreased uptake on the bone marrow scan and abnormal uptake bone scan.
Chambers et al reviewed the charts of 2000 known sickle cell patients. Fourteen patients had an episode of osteomyelitis or septic arthritis. Radiographs and bone scans were not helpful in differentiating infection from an acute bone infarct. Salmonella was the most frequent organism cultured from the osteomyelitis cases. The authors recommend bone aspiration or biopsy in an sickle cell patient with extremity pain, swelling, and a fever greater than 38.2 degrees C.
-
An otherwise healthy, 65-year-old male undergoes a right total knee arthroplasty without complications. Which of the following statements is recommended by the American Academy of Orthopaedic Surgeons Clinical Practice Guidelines for preventing venous thromboembolic disease in patients undergoing elective hip and knee arthroplasty?
- Administer aspirin one week pre-operatively
- Use elastic compressive stockings for 10 days pre-operatively
- Obtain a post-operative duplex ultrasound of the patient for screening purposes
- Administer Coumadin to maintain the patient’s INR between 1.5 and 2.5 during the post-perative period
- Use mechanical compressive devices in the postoperative period
Corrent answer: 5
Use of mechanical compressive devices and aspirin during the postoperative period is
recommended by the American Academy of Orthopaedic Surgeons Clinical Practice Guidelines.
In 2011, the American Academy of Orthopaedic Surgeons published their Clinical Practice Guidelines for preventing venous thromboembolic disease in patients undergoing elective hip and knee arthroplasty. A summary of those guidelines provide general recommendations for venous thromboembolic disease in patients with and without bleeding disorders. These recommendations range from strong (recommending against the use of routine post-operative duplex ultrasonography), to moderate (using mechanical compressive devices or a pharmacologic agent for prophylaxis in the post- operative period), to inconclusive (they are unable to recommend for or against specific pharmacologic prophylaxis agents). Additionally, there are recommendations based on consensus agreement of the authors (the recommendation of early mobilization in the post-operative period).
These guidelines are now much closer to in agreement with the American College of Chest Surgeons (ACCS) 2012 guidelines for VTE prophylaxis. One of the differences between the guidelines is that the AAOS guidelines do not state a specific amount of time that a pharmacologic agent should be given post- operatively while the ACCS recommends such agents for a minimum of 10-14 days.
Incorrect answers:
Answer 1: The AAOS guidelines recommend stopping platelet-inhibiting medications such as aspirin and clopidogrel prior to the procedure.
Answer 2: The AAOS guidelines do not state that using elastic compressive stockings is sufficient for VTE prophylaxis, either pre- or post-operatively. Answer 3: The AAOS guidelines recommend against using routine post- operative duplex ultrasound.
Answer 4: The AAOS guidelines do not make specific recommendations for or against the use of a specific INR range.
-
The definition of effect size is best described as which of the following?
- Likelihood that a statistically significant difference would be found between 2 groups given that a difference truly did exist
- Estimated magnitude of the difference in the means between two groups
- Average of the squares of each value's deviation from the mean
- Range within which it is probable that the true value lies for the whole population of patients
- Probability of obtaining a result equal to or more extreme than what was actually observed assuming the null hypothesis is true
Corrent answer: 2
The effect size is best defined as the magnitude of the difference in the means of the control and experimental groups in a study with respect to the pooled standard deviation. Effect sizes are normally used for continuous variables in contrast to relative risk reduction which is used for dichotomous variables. Power (1), variance (3), confidence interval (4), and P value (5) are the other options provided.
-
Which of the following patients are at greatest risk of having a future vertebral fragility fracture?
- Elderly female with prior hip fragility fracture
- Elderly female with prior distal radius fragility fracture
- Elderly female with prior T6 compression fragility fracture
- Elderly female with a T-score of -3.0
- Elderly female currently on hormone replacement therapy
Corrent answer: 3
History of a prior vertebral fragility fracture is the strongest predictor of a future fragility fracture. A meta-analysis by Klotzbuecher et al examined risk factors for fragility fractures and found an association between prior and subsequent fragility fractures. The strongest associations were observed between prior and subsequent vertebral fractures. They found women with preexisting vertebral fractures had an approximately 4 times greater risk of subsequent vertebral fractures than those without prior fractures. They also found this risk increases with the number of prior vertebral fractures. Other combinations of prior and future fracture sites, such as the hip or wrist, were also strongly associated, but none so high as vertebral fractures.
-
A 79-year-old female falls onto her right hip at home and sustains the injury shown in Figure A. She undergoes an uncemented unipolar hemiarthroplasty. During insertion of the stem into the femoral canal, the patient becomes hypotensive and hypoxic. Which of the following has most likely occurred?
- Femoral shaft fracture
- Inadequate fluid resuscitation during surgery
- Acute myocardial infarction
- Pulmonary embolism caused by dislodging of deep venous thrombosis during hip exposure
- Intramedullary fat and marrow embolization
Corrent answer: 5
During insertion of the femoral stem, the intramedullary pressures are increased. Fat and marrow elements can become embolized into the bloodstream at this point resulting in ventilation perfusion mismatch in the lungs.
Kim et al prospectively followed 156 total hip arthroplasties including bilateral and unilateral procedures as well as cemented and uncemented procedures. They found no difference in fat embolization amongst any of the groups. However, they did find that if patients had evidence of bone marrow cells in the right atrium on the first postoperative day, they developed diffuse encephalopathy with confusion and agitation that lasted for about twenty-four hours.
-
Which of the following investigative studies is most useful in the definitive diagnosis of Amyotrophic Lateral Sclerosis (ALS)?
- Genetic testing
- MRI brain and spinal cord
- Muscle biopsy
- Serum protein electrophoresis and immunoelectrophoresis
- Electrodiagnostic studies
Corrent answer: 5
The diagnosis of ALS requires a period of clinical observation to document the progressive loss of upper and lower motor function. Electrodiagnositic studies are required to make a definitive diagnosis.
Amyotrophic lateral sclerosis (ALS) is the most common degenerative disease of the motor neuron system. Nerve conduction studies and needle electromyography (EMG) are useful for confirming the diagnosis of ALS and for excluding peripheral conditions that resemble ALS. Hallmark findings in the electrodiagnosis of ALS are abnormal motor nerve conduction studies, with normal sensory nerve conduction studies. UMN signs are mild weakness, spasticity, and abnormally brisk reflexes; LMN signs are progressive weakness, wasting, and loss of reflexes and muscle tone.
Brooks et al. developed a diagnostic algorithm for the diagnosis of ALS. The algorithm is based on the degree of certainty of diagnosis, which is increased by the number of body segments that demonstrate upper motor neuron (UMN) and lower motor neuron (LMN) abnormalities. Clinical and electrophysiologic findings in 3 or more body segments is definitive of the diagnosis.
Incorrect Answers:
Answer 1: Genetic testing may be performed to identify genetic defects in some familial types of ALS. However to date, there is no single test that is definitive for diagnosing ALS.
Answer 2: MRI brain and spinal card are used to rule out structural lesions and neurologic conditions that have a slimilar clinical presentation as ALS.
Answer 3: Muscle biopsy is not required in the diagnosis of ALS
Answer 4: Serum protein electrophoresis and immunoelectrophoresis will not provide a definitive diagnosis for ALS. They are used to rule out the differential diagnoses of ALS.
-
Radiographic changes suggestive of osteopetrosis in children are a known complication of which of the following types of medications?
- TNF-alpha inhibitors
- Bone morphogenic proteins
- Bisphosphonates
- Fluoroquinolones
- RANKL antibiodies
Corrent answer: 3
Radiographic changes suggestive of osteopetrosis (marble bone disease) are a known complication of bisphosphonate usage. The common cellular pathway in this process is the osteoclast. Bisphosphonates target and inhibit the osteoclast, and these cells are not functioning in patients with osteopetrosis.
Whyte et al describe a case report of a 12-year-old child with idiopathic hyperphosphatasia treated with bisphosphonates who developed osteopetrosis.
Falk et al report on their small case series which showed the beneficial effects and known complications in the treatment of osteogenesis imperfecta with bisphosphonate therapy.
Marini presents a perspective article describing the off-label use of bisphosphonates in children.
-
What function does computerized physician order entry have on medication monitoring?
- Reduces the rate of medication errors
- Improves physician satisfaction
- Decreases narcotic requirements by patients
- Increases rates of allergy related medication errors
- Improves physician knowledge about the drugs they are prescribing
Corrent answer: 1
Computerized physician order entry has greatest impact on reducing medication errors.
Bobb et al. studied medication errors averted by pharmacists at a 700-bed academic center and concluded that 65% of them would likely have been prevented with computerized physician order entry.
Upperman et al. reviewed medication errors at a pediatric hospital before and after implementation of a computerized physician order entry system. They found a significant decrease in adverse drug events following establishment of the computerized system.
Incorrect Answers:
2: Physician satisfaction has not been correlated with computer entry for medications.
3: Narcotic requirements are related to injury or patient characteristics, not computerized entry.
4: Allergy-related errors are decreased with use of computerized entry.
5: Physician knowledge is not necessarily increased with computerized entry.
-
What is the cellular mechanism of action for non-nitrogen containing bisphosphonates (such as clodronate and etidronate) to induce osteoclast apoptosis?
- Inhibiting caspase
- Inhibiting matrix metalloprotease
- Inhibiting reverse transcriptase
- Targeting of farnesyl diphosphate synthase
- Creating toxic analog of adenosine triphosphate that targets mitochondria
Corrent answer: 5
Bisphosphonates work by one of two mechanisms. Non-nitrogren containing bisphosphonates (such as etidronate and clodronate) work by creating a toxic analog of ATP which inhibits ATP and leads to osteoclast apoptosis. Nitrogen containing bisphosphonates (such as alendronate, risedronate, and zoledronate) inhibit the enzyme farnesyl diphosphate synthase which prevents protein geranylgeranylation and prevents osteoclastic bone resorption.
Reska et. al. specifically discuss the difference between the two pathways. Non-nitrogen containing compounds inhibit ATP production and cause osteoclast apoptosis. Nitrogen containing bisphophonates inhibit protein synthesis by interrupting the cholesterol biosynthetic pathway, which prevents osteoclastic resorption.
-
Which class of antibiotics inhibit early fracture healing through toxic effects on chondrocytes?
- cephalosporins
- quinolones
- penicillins
- macrolides
- sulfonamides
Corrent answer: 2
Animal models have shown that quinolones inhibit early fracture healing through a toxic effect on chondrocytes. The study by Perry et al demonstrated that fracture calluses in the animals treated with quinolones showed a lower histologic grade as compared with control animals representing a less mature callus with the presence of more cartilage and less woven bone. The study by Huddleston et al demonstrated fracture calluses in the animals treated with ciprofloxacin showed abnormalities in cartilage morphology and endochondral bone formation and a significant decrease in the number of chondrocytes compared with the controls. None of the other antibiotics listed are known to have toxic effects on chondrocytes.
-
A mutation of the retinoblastoma gene RB-1 leads to the development of malignancies such as retinoblastoma and osteosarcoma. Which term best characterizes the RB-1 gene?
- growth factor
- retro-oncogene
- proto-oncogene
- oncogene
- tumor suppressor
Corrent answer: 5
A tumor suppressor is a gene whose presence normally prevents neoplasia and whose absence leads to unregulated cell growth. Two well-studied tumor suppressor genes include p53 and RB-1. P53 normally suppresses cell division by blocking the cell cycle if genetic damage is present. If it is absent or mutated, the p53 suppressing effect no longer regulates cell growth and neoplasm results. In a similar fashion, a mutation in RB-1, or the retinoblastoma gene, may leads to retinoblastoma and osteosarcoma.
-
You are counseling a young female patient about her future risk of osteoporosis. Which of the following regarding peak bone mass (PBM) is true?
- PBM is consistently attained by the end of the second decade of life in both men and women.
- PBM is independent from environmental factors.
- PBM correlates strongly with post-menopausal bone mineral density
- Timing of PBM varies based on anatomic site.
- Women attain PBM prior to men, regardless of anatomic site. Corrent answer: 4
The timing and magnitude of PBM varies based on anatomic site. PBM is often reached in the appendicular skeleton earlier than in the axial skeleton.
PBM is defined as the greatest amount of bone an individual will attain in his or her lifetime. Controversy has surrounded the timing of PBM due to significant anatomical variations as well as strong gender-based, genetic, geographic, environmental, and mechanical influences. While women may reach PBM
earlier than men in the hip, the converse has been found to be true of the spine. Furthermore, the age at which each is obtained varies widely. Though the PBM of the hip is most often achieved by the end of the second decade of life, PBM of the spine may not occur until the third or fourth decade of life. Interestingly, PBM has been found to correlate poorly with post-menopausal bone mineral density, likely a result of these strong confounding influences.
Bonjour et al. reviewed the controversy behind peak bone mass. The authors note that the gender-based difference in bone mass becomes expressed during puberty and that there is a large variability in normal values of bone mineral density between anatomic sites.
They conclude that bone mass accumulation can be completed by the end of the second decade at both the lumbar spine and femoral neck, but that this may be significantly influenced by a number of variables during growth such as genetics, diet, endocrine and mechanical factors.
Berger et al. more recently evaluated trends in peak bone mass from longitudinal data in the Canadian Multicentre Osteoporosis Study (CaMos). The authors found that peak bone mass was highly variable between the axial and appendicular skeleton. Specifically, lumbar spine PBM was reached in women
at 33-40 years, but much earlier in men at 19-33 years. Conversely, hip PBM was reached earlier in women at 16-19 years and later in men at 19-21 years. Furthermore, there was a lack of concordance between PBM and BMD over age
65. The authors concluded that there was a high geographic variance and strong confounding environmental influences.
Incorrect answers:
Answer 1: While PMB may be attained by the second decade in life, this is highly variable based on anatomic location and strongly influenced by environmental and genetic factors. While PBM in the hip may frequently be attained by the end of the second decade, that of the lumbar spine is not typically reached until the end of the third decade of life in men and fourth in women.
Answer 2: Environmental factors such as diet, exposures to toxins, and mechanical influences have a strong influence on the age and magnitude of PBM.
Answer 3: There is a lack of correlation between PBM and both the age of onset and severity of osteoporosis, regardless of anatomic location. This is thought to be due to strong genetic and environment influences.
Answer 5: There is significant variability in the age and magnitude of PBM between the
axial and appendicular skeleton. PBM is often attained earlier in women in the appendicular skeleton (the hip for instance) but later in the axial skeleton (the vertebrae for instance).
-
SOX9 is a transcription factor that is a key regulator of which of the following tissues shown in Figures A-E?
- Figure A
- Figure B
- Figure C
- Figure D
- Figure E
Corrent answer: 5
Figure E is a histologic slide of cartilage. Activation of SOX9 leads to increased cartilage matrix production.
Sox9 is a transcription factor which directly regulates the expression of the major proteoglycans and collagens comprising the cartilage extracellular matrix. It is a member of the high mobility group superfamily of nonhistone nuclear proteins. During embryogenesis, SOX9 determines chondrocyte cell fate. It regulates the transcription of type II collagen as it binds to the
promoter and enhancer sequences. SOX9 also enhances the transcription of type IX collagen, type XI collagen, aggrecan, and link protein. These products form the major structural components of the cartilage matrix.
Haudenschild et al. performed a study to determine the factor responsible for the activation of the SOX9 transcription factor. They report that Rho-kinase (ROCK) has profound effects on the actin cytoskeleton, which is instrumental in determining the phenotype and differentiation of chondrocytes. They conclude that SOX9 was found to contain a consensus phosphorylation site for ROCK. They conclude that the interaction between ROCK and SOX9 leads to increased cartilage matrix production.
Hoffman et al. performed a study examining the molecular mechanisms regulating chondroblast differentiation. They report that retinoic acid prevents cells from differentiating into chondroblasts. The inhibition of retinoic acid signaling induces the expression of SOX9, which is a major transcriptional regulator of chondrogenesis. They conclude that retinoid receptor-mediated repression is both necessary and sufficient for chondroblast differentiation.
Figure A is a histologic slide of trabecular bone. Figure B is a histologic slide of adipose tissue. Figure C is a histologic slide of skeletal muscle. Figure D is a histologic slide of a peripheral nerve bundle. Figure E is a histologic slide of articular cartilage.
Incorrect Answers:
Answer 1: Figure A demonstrates trabecular bone. Answer 2: Figure B demonstrates adipose tissue. Answer 3: Figure C demonstrates skeletal muscle. Answer 4: Figure D demonstrates a peripheral nerve.
-
Which of the following factors uses serine-threonine kinase receptors to induce metaplasia of pluripotent mesenchymal stem cells into bone-forming cells?
- Bone morphogenetic protein (BMP)
- Osteoprotegerin (OPG)
- Insulin-like grownth factor 2 (IGF-2)
- Platelet-derived growth factor (PDGF)
- Rank ligand (RANKL)
Corrent answer: 1
Bone morphogenetic proteins (BMPs) use serine-threonine kinase receptors to stimulate proliferation and differentiation of pluripotent mesenchymal stem cells into bone-forming cells.
Bone morphogenetic proteins are a highly studied family of growth factors of bone. With over twenty varieties of BMP, specific actions vary widely with most effects revolving around the development and repair of the musculoskeletal system. In the inflammatory phase of healing BMPs target undifferentiated mesenchymal stem cells through extracellular serine-threonine receptors. Following this extracellular interaction, the signal is transmitted to intracellular molecules called SMADs leading to different cellular responses. BMPs are of great interest given their ability to induce robust bone formation.
Cited concerns regarding BMPs are uncontrolled heterotopic bone formation and cost.
Cheng et al. looked at the osteogenic activity of fourteen different BMPs on mesenchymal progenitor cells. They found BMP-2, 6, and 9 induced high levels of alkaline phosphatase activity in pluripotent stem cells. They conclude BMP-
2, 6, and 9 may play important roles in inducing osteoblast differentiation of mesenchymal stem cells.
Lin et al. reviewed the uses of platelet-derived growth factor, bone morphogenetic proteins, and platelet-rich plasma in foot and ankle surgery. They state that BMPs play a large role in the differentiation and proliferation of pluripotent mesenchymal stem cells to bone-forming cells and cartilage.
Illustration A shows a graphic representation of the role BMPs play in the differentiation of mesenchymal stem cells into bone-forming cells.
Incorrect Answers:
Answer 2: OPG inhibits both osteoclast activation and differentiation by acting as a decoy receptor for RANK-L.
Answer 3: IGF-2 stimulates bone and cartilage formation by acting on a variety of cells through tyrosine kinase receptors.
Answer 4: PDGF uses tyrosine kinase receptors to signal inflammatory cells to migrate to the fracture site.
Answer 5: RANKL stimulates bone resorption through the interaction of RANK receptors on osteoclasts.
-
Which of the following disease-modifying antirheumatic drugs
(DMARDs) acts on the cell membrane receptor illustrated in Figure A?
- Anakinra
- Infliximab
- Rituximab
- Sulfasalazine
- Abatacept
Corrent answer: 1
Anakinra is a disease-modifying antirheumatic drug (DMARD) that acts uniquely as an interleukin IL-1 receptor antagonist that blocks the binding of IL-1 to its cell membrane receptor (Figure A). Its action prevents this pro- inflammatory cascade.
Corticosteroids, biologic agents, tumor necrosis factor (TNF) antagonists (infliximab, etanercept, adalimumab, golimumab and certolizumab), IL-1 antagonists (anakinra), and other DMARDs are all used in the pharmacologic
management of rheumatoid arthritis (RA). Anakinra is considered a 4th line agent in the treatment of RA. Pharmacologic agents are the mainstay of treatment in RA and a more aggressive approach with DMARDs is now favored over a pyramid approach, as this has significantly improved the prognosis of RA.
Pisetsky provided an early overview of the role of TNF blockers in RA, specifically citing infliximab and etanercept. He noted their potent binding to TNF, preventing inflammation by blocking the downstream cascade of this cytokine. He further comments that these agents may be effective in patients who have failed other DMARDs and that their effects may be extended to children.
Howe et al. provided an overview of pharmacologic agents used in RA and provided perioperative recommendations for their use. Perioperative continuation is based on half- life and surgical stress, but the authors recommend continuing methotrexate, sulfasalazine, and hydroxychloroquine in the perioperative period. They conclude that consultation with a
rheumatologist is recommended when feasible, and that the risks and benefits of surgical
complications and RA control must be carefully weighed.
Incorrect Answers:
Answer 2: Infliximab (3rd line) is a human mouse chimeric anti-TNF-alpha monoclonal antibody
Answer 3: Rituximab (other biologic agent) is a monoclonal antibody to CD20 antigen (inhibits B cells)
Answer 4: Sulfasalazine (2nd line) is an anti-inflammatory agent that reduces ESR and CRP, although the exact mechanism is unknown
Answer 5: Abatacept (other biologic agent) is a selective co-stimulation modulator that binds to CD80 and CD86 (inhibits T cells)
-
A pharmaceutical company is marketing a new intra-articular injectable therapy that they claim is superior to intra-articular steroid injection for the treatment of knee osteoarthritis. Patients undergoing this new therapy report a larger improvement in their Knee injury and Osteoarthritis Outcome Score (KOOS) when compared to patients undergoing intra-articular steroid injections. This difference is reported with a p-value of 0.04. Which of the following is true when considering whether or not to use this new therapy on your patients?
- There is no significant difference between therapies, no conclusion can be drawn.
- There is not enough data to determine if the difference in KOOS scores is statistically significantly.
- The difference in KOOS scores between therapies is statistically significant, therefore the new therapy will yield improved clinical outcomes
- A larger sample size is needed to determine if there is truly a difference between the two treatments.
- The minimum clinically important difference must be identified in order to determine the clinical significance of these findings.
Corrent answer: 5
When evaluating the merits of a new intervention, it is critical to determine whether or not a statistically significant difference in outcome between two groups leads to a clinically important difference.
Observed differences between samples reflect either true differences in the populations being compared or are due to random chance (i.e. occasionally random sampling will result in the comparison of two samples that do not reflect true population differences). Statisticians make this determination using p-values. For example, if an observed difference has a reported p-value of
0.05, this means that one would expect to see the observed difference less than 5% of the time if there was no true difference between the data sets. Because this is highly unlikely, p < 0.05 is commonly accepted as the cutoff for statistical significance. In clinical practice however, interventions can produce statistically significant differences without yielding a clinically significant benefit. Thus, the next step is to determine whether or not this statistical difference in KOOS translates to a meaningful clinical difference which would mandate a change in the patient's management.
Fosgate et al. review the effect of sample size on the determination of statistical
significance. The likelihood that a study will yield statistically significant results depends on the chosen sample size. Often, insufficient sample sizes are the result of selecting sample size based on convenience, insufficient funding, and the inefficient utilization of available funding. Type I (alpha) errors refer to the rejection of the null hypothesis when it is true (concluding that a difference exists between two populations when in fact there is none). Type II (beta) errors refer to the non-rejection of the null hypothesis when it is true (concluding that there is no difference between two populations when a difference does exist).
Kocher et al. discuss the limitations of depending on p values alone to determine statistical significance. The commonly accepted significant value of
<0.05 is arbitrary, and while there is little difference between a p-value of
0.49 and 0.51, only the former is considered statistically significant. As an alternative, the biostatistics community has begun to utilize confidence intervals to determine significance. Whereas the p-value is often interpreted as being either significant or not, the 95% confidence interval provides a range of values that allows the reader to interpret the implications of the results.
Incorrect Answers:
Answer 1: There is a statistically significant difference between the reported KOOS scores of the two groups (p =0.04).
Answer 2: The question provides a p-value, which is sufficient to determine statistical significance.
Answer 3: While there is a statistical difference between the two therapies, it is important to determine if a clinical difference exists that would mandate a change in the patient's management.
Answer 4: We do not need a larger sample size, we have achieved statistical significance with the current study.
-
Osteoblasts directly regulate the hematopoietic stem cell population through the Jagged1-Notch pathway. Which of the following activates this pathway?
- Interleukin-1 (IL-1)
- IL-6
- Tumor necrosis factor alpha (TNF-alpha)
- Transforming growth factor beta (TGF-beta)
- Parathyroid hormone (PTH)
Corrent answer: 5
The Jagged1-Notch pathway becomes activated by the action PTH.
Osteoblasts have been found to regulate hematopoietic stem cells and the immune response. They do this through the Jagged1-Notch pathway. PTH induces Jagged1, a membrane-bound protein, on osteoblasts. Jagged1 stimulates Notch receptors on the membrane of hematopoietic stem cells (HSC). This stimulation leads to cell proliferation of HSC.
Lorenzo et al. performed a review of interactions of bone and the immune system. Thy report that stimulation of the PTH receptor on bone cells
increased the number of HSC in bone marrow. This also increased the levels of Jagged1 on osteoblasts. This effect was blocked by using inhibitors of Notch signaling. They conclude that PTH directly stimulates the production of Jagged1 by osteoblasts.
Weber et al. performed a study which demonstrated that activation of the PTH receptor in osteoblasts activates the Jagged1-Notch pathway and expands HSC. They found that PTH treatment increases Jagged1 levels in osteoblastic
cells in vivo and in vitro. They conclude that since Jagged1-Notch signaling has been implicated in HSC interactions and osteoblastic differentiation, this pathway may play a critical role in mediating the PTH-dependent expansion of HSC, as well as the anabolic effect of PTH on bone.
Illustration A is demonstrating PTH's effects. PTH activates numerous cytokines and other mediators on osteoblasts (OBs), osteoclasts (OCs), bone marrow mesenchymal stem cells (MSCs), and hematopoietic stem cells (HSCs) located close to the endosteal niche.
Incorrect Answers:
Answers 1, 2, 3, & 4: The Jagged1-Notch pathway becomes activated by the action of PTH. None of the listed mediators have been implicated in this pathway.
-
A 60-year-old female presents to your office for evaluation of 6 months of back pain. She was referred to you by her primary care physician and presents with a consultation request in hand. You note she was last seen by your partner for hip pain 4 years ago but has not returned to the office since. You perform a detailed history (level 3), comprehensive examination (level 4), and low complexity medical decision making (level 3). You dictate an office note and mail a letter to the referring primary care physician regarding your findings. This encounter should be billed as which of the following?
- Level 3 return
- Level 3 new
- Level 3 consultation
- Level 4 new
- Level 4 consultation
Corrent answer: 3
This encounter should be billed as a level 3 consultation based on the lowest level history, examination, and medical decision making. The patient presented with a referral and a consultation letter was sent to the referring physician.
Billing for office visits can be component based or time-based. Component based billing consists of varying levels of history, examination, and medical decision making. Billing for an encounter is limited by the lowest level of these components. Patients fall into three categories of new, consultation, and
return patients. New patients are those who have not been seen by the physician or someone in the same practice in over 3 years. Consult patients are those who have been referred to the office for advice and not transfer of care with a letter or communication sent to the referring physician at the conclusion of the encounter.
Return patients are those who have been seen within 3 years by the physician or a partner in the practice.
Gill et al. reviewed trends in practice management training in residency. They cite multiple papers demonstrating that residents are not confident in coding abilities (96%), miscode visits (62%), rarely have formal training (87%), and feel programs should implement practice management training (62%). The conclude that practice management should be a part of resident education.
Shalowitz et al. assessed the impact of eliminating consultation codes. They found coding error rates of 32.4% and that changing ambulatory consultation codes to those for new patient visits would save Medicare 534.5 million per year. They conclude that consultation codes are being billed higher than appropriate and, in an effort to reduce costs, these codes should be reevaluated.
Illustration A (Centers for Medicare and Medicaid Services) shows the various coding levels for a new patient (99201-99205 representing Levels 1-5 respectively) and how each level may be determined based on key components as well as time. Illustration B shows the corresponding level of care for new, established, and consultation encounters.
Incorrect Answers:
Answer 1, 2, 4, and 5: This patient falls into the consult category and should be billed as a level 3 based on the lowest level of history, examination, and medical decision making.
-
A 29-year-old female presents to your clinic for surgical evaluation, referred by her family practitioner. She states she has a history of left arm numbness that comes and goes, and was told she had radiculopathy in the past. She recently completed a steroid taper pack, which provided moderate relief. On physical exam, she has decreased sensation in a non-dermatomal distribution involving most of her left arm. Left deltoid strength is 4/5. Left biceps and triceps reflexes are 3+. She has a positive Hoffmann’s sign bilaterally. A radiograph and magnetic
resonance imaging of her cervical spine are shown in Figures A and B, respectively. What is the most appropriate next step in management?
- Plasmapharesis and immunoglobulin administration
- Laminoplasty
- Lumbar magnetic resonance imaging
- Neurology referral
- B12 injections
Corrent answer: 4
In a young female presenting with transient upper extremity neurological symptoms, upper motor signs, and magnetic resonance imaging (MRI) with multiple focal T2- enhancing cord lesions, the most likely diagnosis is multiple sclerosis (MS).
Multiple sclerosis is an inflammatory disorder of the central nervous system causing demyelination and axonal injury. It is capable of producing both motor and sensory dysfunction. It mainly affects young females ages 20-40. Symptoms are most commonly described as remitting and relapsing and may include weakness, paresthesias, falls, incontinence, muscle spasms, fatigue, and optic neuritis. Physical exam may be significant for upper motor findings, muscle weakness, and gait abnormalities. Work-up includes labs, MRI of the brain and spinal cord, and cerebrospinal fluid analysis. Specific MRI findings include multiple sites of focal demyelination and asymmetric periventricular plaques. Corticosteroid are indicated for acute exacerbations.
Kim et al. performed a literature review of 35 studies focusing on the differential diagnosis of cervical spondylotic myelopathy. The most commonly reported differentials included amyotrophic lateral sclerosis (ALS), MS, syringomyelia, and spinal tumors.
They conclude that physical exam findings should be correlated with plain radiographs, MRI, and history to establish the correct diagnosis and that electrodiagnostic studies and cerebrospinal fluid exam should be considered when the diagnosis is equivocal.
Sahraian et al. provided a review article on the role of MRI in the diagnosis and treatment of MS. They comment on the role MRI has played in providing an earlier and more accurate diagnosis, as well as its role in prognostics and as a tool to evaluate the efficacy of experimental research trails. Specific MRI techniques for detecting MS include proton density, T1/T2-weighted images, and fluid-attenuated inversion recovery sequences.
Rovira et al. provided a review article looking at the diagnostic features of MS and the differential diagnosis utilizing spinal MRI. Multiple sclerosis lesions typically are hyper- intense on T2 imaging and are ring enhancing on contrast- enhanced T1 imaging. They state that MS lesions are more commonly found in the cervical spine, typically present in the dorsolateral aspect of the spinal
cord, are short-segment, comprise less than half of the cross-sectional cord area, and are often associated with brain lesions. Other short-segment spinal cord lesions that may mimic MS on MRI include autoimmune disorders, vascular disorders, bacterial and viral infections.
Figure A is a normal cervical spine lateral radiograph. Figure B is a mid-sagittal T2- weighted cervical MRI with multiple focal T2-enhancing lesions. Illustration A is a T2- weighted MRI of the cervical spine demonstrating characteristic lesions of MS. There is one lesion in the dorsal upper cervical spine and one in the upper thoracic spine that are T2-enhancing. These lesions are less than
two vertebral body segment lengths, characteristic of MS lesions.
Incorrect Answers:
Answer 1: This describes the treatment of Guillain-Barre syndrome (GBS). Answer 2: This patient has MS and is not indicated for surgery at this time. Answer 3: While lumbar imaging may also reveal MS lesions, these lesions more commonly present in the cervical spine and additional imaging will not change management at this time.
Answer 5: B12 injections are used in the treatment of subacute combined degeneration of the spinal cord.
-
A 10-year-old male presents to your office with a proximal phalanx fracture. You perform a digital block, closed reduction, and place the patient into a cast. You bill for fracture care and reduction. The patient returns for follow-up at 4- weeks for radiographs and cast removal. At this visit, you perform an extended problem-focused history (level 3), detailed physical exam (level 4), and low-level medical decision making (level 3). You spent 15 minutes of face-to- face time with the patient and his family, half of which was spent on counseling and coordination of care. How should this visit be billed?
- Level 1 established
- Level 2 established
- Level 3 established based on time
- Level 4 established
- This visit should not be billed
Corrent answer: 5
All follow-up related fracture care within 90 days of a procedure is included in a global fee under the "restorative treatment" model for the closed management of fractures. This visit should not be billed separately.
The concept of global payment refers to a "single fee" that is paid for the index surgery/procedure that encompasses all post-operative care within a 90-day window. Code 99024 is used for a postoperative follow-up visit and is associated with a zero-dollar amount. The concept of "restorative treatment" indicates that the physician has performed a manual closed reduction with manipulation, the fracture is placed in acceptable alignment for healing, and that the patient will follow up with that physician for future care.
When a physician provides "restorative treatment", the global fracture care code should be utilized.
For billing and coding purposes, patients are placed in the following categories: new, established, and consultation. The level of evaluation and management services performed is component based (history, physical exam, medical decision making) or time based. Billing must be based on the lowest level of component-based care delivered.
Further information on this topic can be found on the following website provided by the Centers for Medicare and Medicaid Services: https://www.cms.gov/Outreach-and- Education/Medicare- Learning-Network-MLN/MLNProducts/Downloads/eval-mgmt-serv- guide- ICN006764.pdf
Illustration A is a table explaining the different billing levels associated with established patient care. Two of the three (history, physical exam, medical
decision making) must be documented. Alternatively, billing can be time based.
Incorrect Answers:
Answers 1, 2, 3, 4: These visit codes do not apply to post-procedural care within a 90-day global fee window.
-
Which of the following is the correct cell lineage and mediator of differentiation of the cell identified by the black arrow in Figure A?
- Myeloid lineage/receptor activator of nuclear factor kappa-B ligand (RANKL)
- Myeloid lineage/bone morphogenetic protein (BMP)
- Lymphoid lineage/receptor activator of nuclear factor kappa-B ligand (RANKL)
- Lymphoid lineage/bone morphogenetic protein (BMP)
- Lymphoid lineage/osteoprotegerin (OPG)
Corrent answer: 1
The osteoclast is derived from a myeloid lineage and differentiation is mediated by receptor activator of nuclear factor kappa-B ligand (RANKL).
The osteoclast is derived from a myeloid hematopoietic precursor cell. This pathway is of the monocyte/macrophage lineage and includes fusion of mononuclear cells into multinucleated osteoclasts. Differentiation is mediated by a combination of macrophage colony-stimulating factor (M-CSF) and RANKL. Osteoclast progenitor cells express RANK receptor on their membrane which allows for differentiation signaling.
Zaidi et al. reviewed recent literature surrounding signaling of osteoclasts. They discuss the various transcription factors and molecular molecules involved in the pathway of osteoclastogenesis. They conclude if any of the
genes encoding critical molecules that regulate osteoclastogenesis are deleted, the phenotype of osteopetrosis is almost always present.
Goodman et al. reviewed the effects of various medications on bone. They state corticosteroids increase production of RANKL and decreases osteoprotegerin (OPG) resulting in increased osteoclastogenesis. They conclude that corticosteroids increase osteoclast formation as well as osteoclast longevity leading to overall increased osteoclastic bone resorption.
Figure A shows a histologic specimen of bone with a black arrow identifying an osteoclast. Illustration A shows a depiction of the osteoclast lineage and associated mediators.
Illustration B shows the myeloid lineage of the osteoclast and the relationship to other cell types.
Incorrect Answers:
Answer 2: Bone morphogenetic proteins (BMPs) stimulate proliferation and differentiation of pluripotent mesenchymal stem cells into bone-forming cells. Answer 3 & 4: Lymphoid lineage leads to precursors of adaptive immunity such as T cells, B cells, and natural killer cells.
Answer 5: Osteoprotegerin downregulates osteoclastogenesis by acting as a decoy for RANKL and inhibiting RANKL binding with RANK receptors on osteoclasts.
-
What does a power analysis determine and when should it be performed?
- Sample size; prior to initiating the study
- Effect size; after the study is completed
- Type-I error; after the study is completed
- Type-II error; after the study is completed
- Number needed to treat; prior to initiating the study
Corrent answer: 1
A power analysis determines sample size and should be performed prior to initiating a study.
Power is the probability of finding a significant association if one truly exists. It is defined as 1 - the probability of a type 2 error (beta). Usually, power is set
at > 80%, which means that there is a <20% chance that the study will demonstrate no significant association when one exists. When a study demonstrates no significant association, there is a concern for type-II (beta) error as expressed by power. In cases such as this, the sample size may be too small. To determine sample size a power analysis is performed. There are four elements involved in a power analysis: alpha, beta, effect size, and
sample size. Typically, power is set at 80%, alpha is set at 0.05, the effect size and variance are estimated from pilot data or prior literature, and the equation is solved for the
necessary sample size. Calculation of power after a study has been completed is controversial and discouraged.
Kocher et al. performed a review of clinical epidemiology and biostatistics as a primer for orthopedic surgeons. They report that a power analysis should be performed prior to initiating the study. This will ensure the study is appropriately powered and decrease the effects of chance. They conclude that when a study determines no significant effect the power of the study should be reported.
Fosgate performed a review of practical sample size calculations for surveillance and diagnostic investigations. He reports that the likelihood that a study will yield statistically significant results depends on the chosen sample size. He lists reasons for inadequately sized studies that do not achieve statistical significance include failure to perform a power analysis, selecting sample size based on convenience, insufficient funding, and inefficient utilization of available funding. He concludes that sample sizes should be increased as some test subjects are expected to be lost.
Incorrect Answers:
Answer 2: The effect size is defined as the magnitude of a difference considered to be clinically meaningful. It is used in the power analysis to determine the required sample size.
Answer 3: Type I error, or alpha error, is when the null hypothesis is rejected even though it is true. A significant association is found when none truly exits. The alpha level is the threshold of statistical significance established (p-value
< 0.05 by convention).
Answer 4: Type II error, or beta error, is an error where no significant association is found when one truly exists. The null hypothesis is accepted when it is false and should be rejected.
Answer 5: Number needed to treat is the number of patients that must be treated in order to achieve one additional favorable outcome. It is calculated by 1 / absolute risk reduction.
-
Which of the following is involved in the process of osteoinduction and osteogenesis?
- Bone morphogenic protein-1 (BMP-1)
- BMP-2
- BMP-3
- Peroxisome proliferator-activated receptor gamma (PPAR-Gamma)
- Insulin-like growth factor 1 (IGF-1)
Corrent answer: 2
Bone morphogenic protein-2 is directly responsible for osteoinduction and osteogenesis.
Bone morphogenic proteins were discovered as the bone-forming factors in demineralized bone used in surgical procedures. BMPs have therefore found a role in bone regeneration strategies as they have been found to initiate the complete cascade of
bone formation. As members of the transforming growth factor-beta (TGF-beta) superfamily, BMPs play important roles in skeletal development and bone formation. BMPs initiate their signaling transduction by binding to a heterodimeric complex of two transmembrane serine-threonine kinase receptors, BMP receptor (BMPR) type I and BMPR type I. The activated receptor kinases, in turn, phosphorylate the transcription factors Smad 1, 5, and 8. The phosphorylated Smads then form a heterodimeric complex with Smad 4 in the nucleus and activate the expression of target genes in concert with other coactivators. BMP-2, specifically, exhibits potent osteogenic and osteoinductive activity.
Cheng et al. performed a study to determine the activity of the 14 known types of BMP in mesenchymal progenitor and osteoblastic cells. Osteogenic activity was determined by measuring the induction of alkaline phosphatase. They found that BMP-2, 6, and 9 significantly induced alkaline phosphatase activity in pluripotent cells while BMP-2, 4, 6, 7, and 9 significantly induced alkaline phosphatase activity in preosteoblastic cells. BMP-3 was found to have no activity. They conclude that BMP-2, 6, and 9 may play an important role in inducing osteoblast differentiation of mesenchymal stem cells.
Kempen et al. performed a study to determine if the sequential release of vascular endothelial growth factor (VEGF) and BMPs could enhance BMP-2- induced bone formation. A microsphere composite of BMP-2 embedded in a propylene scaffold surrounded by a hydrogel loaded with VEGF was implanted in rats. Empty microspheres or microspheres with either VEGF or BMP were used as controls. They found that in combination with local sustained BMP-2 release, VEGF significantly enhanced ectopic bone formation compared to BMP alone. They conclude that sequential angiogenic and osteogenic growth factor release may be beneficial in bone regeneration.
Illustration A (Cheng et al.) is a figure demonstrating the distinct osteogenic activity of human BMPs. BMP-2, 6, and 9 are the most potent agents to induce osteoblast lineage differentiation of mesenchymal progenitor cells while most
BMPs can promote the terminal differentiation of committed osteoblast precursors.
Incorrect Answers:
Answer 1: BMP-1 is a metalloprotease which is a protease for types I, II, and III collagen. It is unable to induce bone formation.
Answer 3: BMP-3 does not exhibit osteogenic or osteoinductive ability. Answer 4: PPAR-gamma is an adipogenic factor of mesenchymal stem cells. Answer 5: IGF-1 is the most abundant growth factor in bone. It induces proliferation without maturation of the growth plate and is necessary for skeletal growth.
-
An investigator is conducting a randomized, controlled trial to determine if there are any differences in patient-reported outcomes (PROs) between two implants used to treat displaced proximal humerus fractures. An a priori power analysis reveals that 84 patients will be required in each arm to show a difference between groups with a beta of 0.2 and an alpha of 0.05. After enrolling 88 patients in each arm, the results are analyzed and no difference (p=0.28) in PROs are found between the two groups. What is the chance that the null hypothesis is accepted when it is actually false?
1. 0.8
2. 0.2
3. 0.05
4. 0.28
5. 0.95
Corrent answer: 2
The chance that the null hypothesis is accepted when it is actually false is 0.2.
A type II error, also called a beta error, occurs when the null hypothesis is accepted when it is actually false (a false negative finding). A type II error can
occur when a study is underpowered, but even if a study is adequately powered it can occur by chance. The chance of a type II error occurring is beta, which can be set during a power analysis. A type I error also called an alpha error occurs when the null hypothesis is rejected when it is actually true (a false positive finding). The chance of a type I error occurring is equal to alpha, which is usually set at 0.05, or 1/20, by convention.
The review article by Kocher et al. provides a brief but comprehensive overview of power analyses, type I error, type II error, and the concept of statistical power. The authors point out that beta represents the chance of a type II error, while alpha represents the chance of a type I error, and that conventionally beta is set at 0.2 and alpha at 0.05. The authors also state that when a study finds no difference, the power of the study, or 1-beta, should be reported.
Lochner et al. analyzed 19,942 patients from 117 randomized studies in orthopaedic trauma. The authors found that the mean overall study power was
24.65% and that the type II error rate for primary outcomes was 90.52%. The authors concluded that investigators can reduce type II error rates by performing power analyses prior to beginning a trial.
Incorrect Answers:
Answer 1: This reflects the power of the study, 1 - beta, or 0.8. Answer 3: The chance of an alpha error is 0.05 (1/20).
Answer 4: This is the p-value for the difference in outcomes between the two groups in the study. At an alpha of 0.05, the p-value of 0.28 would be deemed insignificant.
Answer 5: This is 1 - alpha, and does not represent type II error or power.
-
What is the role of matrix metallopeptidase-13 (MMP-13) in the early callus phase of bone healing?
- Expressed by terminally differentiated chondrocytes to degrade the cartilaginous extracellular matrix
- Expressed by immature chondrocytes to degrade the calcified extracellular matrix
- Expressed by terminally differentiated chondrocytes to degrade the calcified extracellular matrix
- Expressed by terminally differentiated osteoclasts to degrade the calcified extracellular matrix
- Expressed by terminally differentiated osteoclasts to degrade the cartilaginous extracellular matrix
Corrent answer: 1
During the early callus phase of bone healing, MMP-13 is expressed by terminally differentiated chondrocytes to degrade the cartilaginous extracellular matrix.
MMPs are a family of zinc-dependent proteolytic enzymes that can degrade many protein components of the extracellular matrix. Their activity is necessary for matrix turnover during embryogenesis, morphogenesis, normal tissue remodeling, and repair. MMP-13 belongs to the collagenase subgroup of the MMP family as it is able to cleave interstitial fibrillar collagens. It is constitutively produced by terminally differentiated chondrocytes and, in the normal state, it is rapidly endocytosed and degraded. In the presence of a fracture, MMP-13 degrades the collagenous extracellular matrix in the early callus phase and assists in converting the soft callus into woven bone. It's aberrant activity has been implicated in arthritis, cancer, atherosclerosis, and fibrosis.
Gerstenfeld et al. performed a study to develop three-dimensional reconstructions of fracture callus morphogenesis. They collected rat and mouse femur and tibia fracture calluses over various time points of healing. They found that endochondral bone formation occurs asymmetrically with cartilage tissues seen proximal or distal to the fractures in the callus. Remodeling of the calcified cartilage proceeded from the edges of the callus inwards producing a supported trabecular structure over which a thin outer cortex formed. They concluded that remodeling of calcified cartilage produces a trabecular bone structure which provides rapid increases in weight bearing capacity.
Gerstenfeld et al. performed a study to determine if the inhibitory effects of cyclooxygenase-2-(COX-2)-specific anti-inflammatory drugs and nonsteroidal anti- inflammatory drugs (NSAIDs) are reversible after short-term treatment. They administered the medications orally to rats for either 7 or 21 days and assessed fracture-healing with biomechanical, histological, and biochemical analyses. They found that COX-2-specific drugs inhibit fracture-healing more than NSAIDs, and the effect is related to the duration of treatment. They conclude that reduced prostaglandin levels in callus rebound with drug withdrawal, and impairment in the mechanical integrity of fractures is reversed after short- term treatment.
Illustration A demonstrates the stages of fracture healing.
Incorrect Answers:
Answer 2: MMP-13 is produced by terminally differentiated chondrocytes, not immature chondrocytes.
Answer 3: MMP-13 degrades cartilaginous extracellular matrix, not calcified extracellular matrix.
Answers 4 & 5: MMP-13 is produced by terminally differentiated chondrocytes, not osteoclasts.
-
A 68-year-old woman presents to review the results of a dual- energy x-ray absorptiometry scan. She was found to have a T-score of
-2.6. When discussing treatment options, the patient mentions that she saw a commercial advertising a medication that builds new bone. Which of the following is the correct description of this drug?
- A protein that is produced by osteocytes and is encoded by the SOST gene.
- A recombinant form of a hormone that is produced by chief cells
- A competitive partial agonist of the estrogen receptor
- A polypeptide hormone produced by parafollicular cells
- A drug which contains a P-C-P backbone
Corrent answer: 2
Parathyroid hormone (PTH) is a hormone which is produced by chief cells in
the parathyroid gland. Teriparatide is a recombinant protein form of PTH and is the only available anabolic agent in the United States.
PTH stimulates bone formation and resorption and can increase or decrease bone mass, depending on the mode of administration. Teriparatide or PTH (1-
34), comprises the first 34 amino acids of the hormone and produces its chief biologic effects. Continuous infusions which will result in a persistent elevation of serum PTH concentration will lead to greater bone resorption and hypercalcemia compared to daily injections. Daily injections lead to only
transient increases of PTH which may stimulate bone formation. Intermittent administration increases the number of osteoblasts, activation of pre-existing osteoblasts, increased differentiation of lining cells to become osteoblasts, and reduced osteoblast apoptosis.
Neer et al. performed a study to determine the effect of PTH for the treatment of postmenopausal women with prior vertebral fractures. They randomly assigned 1637 postmenopausal women with prior vertebral fractures to
receive 20 or 40 µg of PTH (1-34) or placebo administered daily. They performed serial measurements of bone mass by dual-energy x-ray absorptiometry. They found new vertebral fractures in 14% of the placebo group compared to 5% in the treatment group. Compared with placebo, PTH increased bone mineral density by 9-13%. They conclude that treatment with PTH (1-34) decreases the risk of vertebral fractures and increases bone mineral density.
Deal wrote a review on the use of intermittent human PTH in the treatment of osteoporosis. He reports that biosynthetic human PTH (1-34) is also known as teriparatide. Its bioavailability is approximately 95% after subcutaneous administration and maximum serum levels are achieved after approximately
30 minutes. It is metabolized in the liver and kidney and no clinically important interactions with other drugs have been found. He reports that in a rat toxicology study in which PTH was administered in high doses of an extended period of time, osteosarcoma was seen. None of the patients in clinical trials have developed osteosarcoma, however.
Incorrect Answers:
Answer 1: A protein that is produced by osteocytes and is encoded by the SOST gene describes sclerostin. Sclerostin decreases bone mass by inhibiting the Wnt pathway. Answer 3: A competitive partial agonist of the estrogen receptor describes raloxifene. Raloxifene slows bone resorption and is not anabolic.
Answer 4: A polypeptide hormone produced by parafollicular cells describes calcitonin. Calcitonin is produced by the thyroid and binds membrane receptors on osteoclasts to inhibit resorption. It is not anabolic.
Answer 5: A drug which contains a P-C-P backbone describes a bisphosphonate. Bisphosphonates inhibit osteoclast resorption and are not anabolic.
-
A patient previously diagnosed with hypovitaminosis D follows up for repeat laboratory values following initiation of repletion therapy. In the pathway shown in Figure A, what letter corresponds to the serum marker commonly obtained in this scenario?
- A
- B
- C
- D
- E
Corrent answer: 2
Letter B corresponds to 25-hydroxyvitamin D, the preferred laboratory assay for evaluating vitamin D deficiency.
Vitamin D has two bioequivalent forms; D2 (ergocalciferol) is from vegetables and oral supplements and D3 (cholecalciferol) is from UVB radiation/sunlight, oily fish, food (milk, juices, margarines, yogurts, cereals, and soy), and oral supplements. Both D2 and D3 are biologically inert and are metabolized in the liver to 25-hydroxyvitamin D (calcidiol, which includes 25-hydroxy D2 and 25- hydroxy D3). Calcidiol is converted to 1,25-hydroxy (calcitriol) in the kidney
by 1α-hydroxylase. For measuring serum levels, 25-hydroxy is the best indicator of vitamin D stores because it has half-life of 15 days; 1, 25- dihydroxy has a half-life of 15 hours, and only a fraction of 25-hydroxy is converted to 1,25 dihydroxy. The recommended daily allowance of Vitamin D3 for patients ages 1-70 years old is 600 International Units (IU) (both male and female, lactating/pregnant and non-lactating/pregnant), 400 IU for infants < 1 year old, and 800 IU for adults > 70 years old.
Norman et al. provide an overview of the role and metabolic pathways involving vitamin D, and also include a historical perspective. They emphasize its importance in calcium and phosphorous regulation. They also note its role in rickets and the discovery of ultraviolet light in cholecalciferol's metabolic pathway.
Patton et al. provide a general review article commenting on the role of vitamin D in orthopaedics. They emphasize its role in bone healing, neuromuscular function, and musculoskeletal development. They conclude that many orthopaedic patients may be at risk for vitamin D deficiency, which may increase their risk of fracture and impair bone healing.
Figure A is a diagram of the metabolic pathway involving cholecalciferol, and Illustration A is the completed diagram shown in Figure A.
Incorrect Answers:
Answer 1: Letter A corresponds to vitamin D3 (cholecalciferol) Answer 3: Letter C corresponds to 1,25(OH)2D3 (calcitriol) Answer 4: Letter D corresponds to 24,25(OH)2D3
Answer 5: Letter E corresponds to 1,24,25(OH)3D3
-
The seventh character in the ICD-10 coding system describes which of the following?
- Site of injury
- Laterality of injury
- Etiology of injury
- Category of pathology
- Phase of treatment
Corrent answer: 5
The seventh character in the ICD-10 coding system is used to relay information about the phase of treatment.
ICD-10-CM is a clinical modification of the World Health Organization’s ICD-10, which consists of a diagnostic system. ICD-10-CM includes the level of detail needed for morbidity classification and diagnostic specificity. It also provides
code titles and language that complement accepted clinical practice. As with ICD-9- CM, ICD-10-CM is maintained by the National Center for Health Statistics.
The system consists of more than 68,000 codes, compared to approximately
13,000 ICD-9-CM codes. ICD-10-CM codes have the potential to reveal more about quality of care so that data can be used in a more meaningful way to better understand complications, better design clinically robust algorithms, and better track the outcomes of care. ICD-10-CM incorporates greater specificity and clinical detail to provide information for clinical decision making and outcomes research.
The ICD-10 system consists of 3-7 characters: the first character is alpha, the second is numeric, and the third through seventh are alpha or numeric.
The seventh character can be listed as any of the following: A - Initial encounter for closed fracture
B - Initial encounter for open fracture type I or II
C - Initial encounter for open fracture type IIIA, IIIB, or IIIC D - Subsequent encounter for fracture with routine healing
E - Subsequent encounter for open fracture type I or II with routine healing
F - Subsequent encounter for open fracture type IIIA, IIIB, or IIIC with routine healing G - Subsequent encounter for fracture with delayed healing
H - Subsequent encounter for open fracture type I or II with delayed healing J -
Subsequent encounter for open fracture type IIIA, IIIB, or IIIC with delayed healing K - Subsequent encounter for fracture with nonunion
M - Subsequent encounter for open fracture type I or II with nonunion
N - Subsequent encounter for open fracture type IIIA, IIIB, or IIIC with nonunion P - Subsequent encounter for fracture with malunion
Q - Subsequent encounter for open fracture type I or II with malunion
R - Subsequent encounter for open fracture type IIIA, IIIB, or IIIC with malunion S - Sequela
Illustration A shows the overall ICD-10 coding system format.
The referenced instructional course lecture by Bert et al. notes that the ICD- 10-CM allows for more specific descriptors of a procedure and is broken down
by category, etiology, anatomic site, severity, and extension, and contains 4.9 times more codes than ICD-9.
-
A new patient presents to your office with a one-week history of lower back pain. English is her second language. She endorses radicular-type pain radiating down her left thigh without motor weakness and specifically denies saddle anesthesia or incontinence. Her physical exam is benign other than lumbar paraspinal tenderness and a positive straight leg raise test on the left. In this encounter, which of the following interventions may have the effect of increasing both patient satisfaction and the quality of care provided?
- Prescribing hydrocodone to a patient at their request
- Ordering an MRI
- Providing an interpreter to a patient with limited English proficiency
- Ensuring that the waiting room is supplied with adequate reading material and beverages
- Referral to pain management for diagnostic and therapeutic lumbar injections
Corrent answer: 3
Providing an interpreter to a patient with limited English proficiency will likely improve both patient satisfaction and the quality of care delivered.
Several measures have been developed to assess patient-centered care and the patient experience. As hospitals are now being measured, ranked, and reimbursed based partially on patient satisfaction, increased resources have been allocated to
improve and study these metrics. One such patient satisfaction survey is the Hospital Consumer Assessment of Healthcare Providers and Systems (HCAHPS), designed to evaluate and compare the inpatient experience in U.S. hospitals. However, there is concern that these patient-reported measures do not accurately reflect the quality of care delivered and may have several adverse effects.
Shirley et al. describe the relationship between quality of care and patient satisfaction scores. They state that, although quality comprises such factors as safety, efficiency, outcomes, and patient experience, patient satisfaction surveys incorporate factors outside of these domains that may impede the quality of care delivered and increase costs in an effort to boost satisfaction metrics. They conclude that satisfaction surveys are not appropriate measures of overall healthcare quality.
Godil et al. performed a prospective cohort study including 58 patients undergoing transforaminal lumbar interbody fusions for degenerative lumbar spondylolisthesis in order to determine the most valid and responsive instruments to assess quality and effectiveness. Measures to assess pain, disability, general health, quality of life, and depression were utilized. They concluded that the Oswestry Disability Index was the most valid and
responsive measure for assessing pain and disability, whereas the EuroQol five dimensions was the most valid and responsive measure of improvement for health-related quality of life.
Manary et al. provide a perspective piece on the patient experience and health outcomes. They raise concerns that, although such measures are increasingly utilized in research and policy, their accuracy in quality assessment has not been proven. Despite this, the authors feel that available evidence supports that survey content, risk adjustment, and timing of survey administration utilized in surveys such as HCAHPS does correlate with healthcare quality.
Incorrect Answers:
Answer 1: Prescribing medications requested by patients will likely increase patient satisfaction scores but not improve the quality of care provided; narcotics are not indicated in the first-line treatment of acute low back pain. Answer 2: Obtaining imaging, especially when requested by the patient, may improve patient satisfaction but not necessarily the value or quality of care provided; MRI is not indicated initially in the scenario described.
Answer 4: While patient satisfaction ratings may include cleanliness and facility features, these have little to do with the value or quality of care delivered.
Answer 5: A referral to pain management for injections is not indicated acutely in the scenario described, but may be considered, along with advanced imaging, if the patient does not improve with therapy, activity modification,
and non-narcotic analgesics.
-
An orthopaedic surgeon published an article on double-bundle ACL reconstruction. He describes his current surgical technique developed over the course of 25 years of experience. Which of the following is
the most accurate representation of this study's level of evidence?
- Therapeutic study, Level III
- Prognostic study, Level III
- Therapeutic study, Level IV
- Prognostic study, Level IV
- Level V
Corrent answer: 5
This study is based solely on one surgeon's expert opinion which indicates a Level V study.
Levels of evidence are assigned to determine the clinical value of a study. A Level V study is the lowest level of evidence assigned and is based on either a case report, a personal observation, or an expert opinion. Level I studies are randomized controlled trials or meta-analyses of randomized trials with homogeneous results. Level II studies are prospective cohort studies or meta- analyses of prospective cohort studies. Level III studies are retrospective cohort studies or case-control studies or meta-analyses of Level III studies. Level IV studies include case series.
Wright et al. performed a review of the grades of evidence assigned to studies. They report that levels of ratings for multiple studies addressing a clinical
grade recommendation are summarized with the use of a grade of recommendation. Grade A recommendations are based on consistent Level I studies. Grade B recommendations are based on consistent Level II or III evidence. Grade C recommendations represent either conflicting evidence or are based on Level IV or V evidence. A grade of I indicates that there is insufficient evidence to make a treatment recommendation.
Wright et al. published a table detailing levels of evidence for orthopaedic studies. They define a Level V study as an expert opinion.
Illustration A is a table detailing the levels of evidence for orthopaedic studies (Wright et al.). Illustration B is a table detailing the grades of evidence (Wright et al.).
Incorrect Answers:
Answer 1: A therapeutic study investigates the results of treatment. A Level III study is a case-control study, a retrospective cohort study, or a systematic review of Level III studies.
Answer 2: A prognostic study investigates the outcome of disease.
Answers 3 & 4: A Level IV study is a case series with no historical or control group.
-
A patient presents with the clinical images shown in Figures A-C. Which of the following is true?
- Most patients with this skin condition develop arthritis
- All patients that develop arthritis are positive for HLA-B27
- The skin lesions respond well to systemic anti-fungal therapy
- Rheumatoid factor and ANA are usually positive
- DIP erosions are often centripetal
Corrent answer: 5
The clinical presentation is consistent with psoriatic arthritis. DIP joint involvement is characterized by centripetal erosions (pencil-in-cup deformity).
Psoriatic arthritis is one of the seronegative spondyloarthropathies. As such, it most commonly will have have a negative rheumatoid factor and a predilection for patients with the HLA-B27 gene. The majority of patients with psoriasis do not develop psoriatic arthritis. For those that do develop arthritis, the hands and feet are often involved with the DIP joints classically affected. The centripetal erosions and classic pencil-in-cup deformity help differentiate it
from osteoarthritis. These patients require referral to and management by a rheumatologist, as first-line therapies are medically based.
Choo et al. review inflammatory arthroses of the hand and wrist. They state that the management of these and several other conditions is typically medical in nature and continues to evolve with the development of biologically targeted medications. Surgical treatment is infrequently undertaken but can be efficacious for severe cases to alleviate symptoms and for deformity
correction.
Taylor et al. discuss the development of a new classification system for psoriatic arthritis. The CASPAR (ClASsification criteria for Psoriatic ARthritis) criteria is a scoring system based on some of the most common manifestations.
Figures A, B, and C are clinical pictures of a psoriatic plaque, nail pitting, and dactylitis, respectively. These are classic manifestations of psoriatic arthritis.
Illustration A shows the classic centripetal erosion in a left hand with psoriatic arthritis. Due to the erosions and osteolysis, the joint space often widens and deformity may develop. Illustration B is the CASPAR criteria for the diagnosis of psoriatic arthritis.
Incorrect Answers:
Answer 1: Only 5-20% of patients with psoriasis develop psoriatic arthritis. Answer 2: Approximately 50% of patients with psoriatic arthritis are HLA-B27 positive.
Answer 3: The skin lesions are not a result of a fungal infection and are therefore not treated with anti-fungal medications.
Answer 4: RF and ANA are usually negative in psoriatic arthritis.
FOR ALL MCQS CLICK THE LINK ORTHO MCQ BANK